Re: [obm-l] Problema!

2007-09-03 Thread Rogerio Ponce
Ola' pessoal,

inicialmente, durante M horas (correspondentes a meio dia) , N pessoas 
trabalharam no campo grande. Em seguida, durante M horas, N/2 pessoas ainda 
trabalharam no campo grande. Enquanto isso, no campo pequeno, N/2 pessoas 
trabalharam durante M horas, e, no dia seguinte, 1 pessoa trabalhou por M horas.

Entao, como o trabalho total realizado no campo grande deve ser o dobro do 
trabalho realizado no campo pequeno, temos que:
N * M + N/2 * M = 2 * ( N/2 * M + 1 * M )
de onde N=4

Portanto, havia 4 trabalhadores no grupo.

[]'s
Rogerio Ponce
---
   
Um  grupo de trabalhadores tinha a tarefa de realizar a colheita de 
dois  campos de trigo, um dos quais tinha o dobro da área do outro. 
Durante meio dia todo o pessoal do grupo trabalhou no campo de trigo grande.

Depois do almoço, metade do pessoal continuou no campo de trigo grande e a  
outra metade trabalhou no campo de trigo pequeno. Durante a tarde acabaram 
ambos as tarefas exceto uma pequena área  do campo  de trigo menor, em cuja 
colheita se ocupou todo o dia seguinte apenas um  trabalhador. 
Quantos trabalhadores haviam neste grupo?

Claudio.



   Flickr agora em português. Você clica, todo mundo vê. Saiba mais.

Re: [obm-l] Problema

2007-11-20 Thread saulo nilson
2 -35 - 48
2 -5-  -3-  x
1 -1 - -9- 45
T*(k1P+k2A)=N
3*(2k1+k2*5)=48
9(k1+k2)=45
2k1+5k2=16
2k1+2k2=10
k2=2
k1=3
N=5(6+6)=60


On 11/16/07, Antonio Manuel Castro del Rio <[EMAIL PROTECTED]>
wrote:
>
> Alguém teria a solução do seguinte problema.
>
> Dois profissionais e cinco aprendizes, produzem 48 peças em 3 dias; um
> profissional e um aprendiz produzem 45 peças em 9 dias. Quantas peças são
> produzidas por 2 profissionais e 3 aprendizes em 5 dias? Resposta 60 peças.
>
>
>


Re: [obm-l] Problema!!

2009-04-13 Thread Paulo Santa Rita
Ola Antonio e demais
colegas desta lista ... OBM-L,
(escreverei sem acentos)

E bem conhecido que " um numero natural  pode ser escrito como soma de
dois quadrados se, e somente se, na sua decomposicao em fatores primos
os fatores da forma 4N+3 tenham expoente par ". Como 96=(2^5)*3, ve-se
que o fator primo 3 ( que e da forma 4N+3 ) nao tem expoente par.
Logo, o numero 96 nao pode ser representado como soma de dois
quadrados.

Este tema da representacao de numeros como soma de dois quadrados e
bem conhecido e, em geral,  abordado nos cursos iniciais sobre teoria
dos numeros.

um abraco a todos
PSR, 21304091042




Um n

2009/3/24 Antonio Manuel Castro del Rio :
> Ola, boa noite.
> Preciso de ajuda para resolver um problema.
>
>   COMO FAZER 96 VIRAR UMA SOMA DE DOIS QUADRADOS?
>
> Desde já, obrigado
>    Antonio del Rio
>
>

=
Instru��es para entrar na lista, sair da lista e usar a lista em
http://www.mat.puc-rio.br/~obmlistas/obm-l.html
=


Re: [obm-l] Problema

2002-12-17 Thread Cláudio \(Prática\)
8 escrivaninhas certamente são suficientes.

Se cada professor usar uma escrivaninha 90% do tempo, então o número total
de "escrivaninhas-tempo" utilizadas será igual a 8 * 0,90 = 7,2. Assim,
supondo que não exista 0,2 escrivaninha, 8 escrivaninhas são também
necessárias.

Um abraço,
Claudio Buffara.

- Original Message -
From: "Tertuliano Carneiro de Souza Neto" <[EMAIL PROTECTED]>
To: <[EMAIL PROTECTED]>
Sent: Tuesday, December 17, 2002 2:48 PM
Subject: [obm-l] Problema


Alguém pode me ajudar no seguinte problema?



O departamento de matemática tem 8 professores
graduados ocupando o mesmo gabinete. Cada um tanto
estuda em casa como no gabinete. Quantas escrivaninhas
deve haver no gabinete de modo que cada um tenha uma
pelo menos 90% do tempo?

Grato.
Tertuliano Carneiro,
De Salvador.

___
Yahoo! GeoCities
Tudo para criar o seu site: ferramentas fáceis de usar, espaço de sobra e
acessórios.
http://br.geocities.yahoo.com/
=
Instruções para entrar na lista, sair da lista e usar a lista em
http://www.mat.puc-rio.br/~nicolau/olimp/obm-l.html
O administrador desta lista é <[EMAIL PROTECTED]>
=

=
Instruções para entrar na lista, sair da lista e usar a lista em
http://www.mat.puc-rio.br/~nicolau/olimp/obm-l.html
O administrador desta lista é <[EMAIL PROTECTED]>
=



Re: [obm-l] Problema

2003-01-13 Thread Rodrigo Villard Milet
Se você sabe um pouco de álgebra linear fica fácil. Seja k o menor natural
tal que A^k = 0. Portanto, existe um vetor v de R^n tal que A^(k-1) * v não
é zero. Agora provamos que v, Av, ... , A^(k-1) * v são um conjunto l.i.
Suponha que temos a(0)*v + a(1)*Av + ... + a(k-1)*A^(k-1) * v = 0, com a(i)
reais. Multiplique essa equação por A^(k-1) à esquerda, daí segue que
a(0)=0. Depois multiplique por A^(k-2) e terá que a(1)=0. Dessa mesma forma,
mostramos que a(0)=a(1)=...=a(k-1)=0, logo o conjunto é l.i. Isso prova em
particular que k<=n, pois não podemos ter mais de n vetores l.i em R^n.
Voltando ao seu problema se temos que A^(n+1)=0 é pq n+1 não pode ser o
menor número k tal que A^k=0 (pelo OBS acima). Então k<=n. Se k=n, acabou,
se k é menor que n, segue que A^n = A^k * A^(n-k) = 0.
Talvez tenha um jeito mais simples pra fazer isso..
Abraços,
 Villard

-Mensagem original-
De: [EMAIL PROTECTED] <[EMAIL PROTECTED]>
Para: [EMAIL PROTECTED] <[EMAIL PROTECTED]>
Data: Segunda-feira, 13 de Janeiro de 2003 18:34
Assunto: [obm-l] Problema


>
>OLa galera,
>
>Estou enviando um bom problema de matriz. La vai...
>Seja  A uma matriz nxn. Prove que se A^(n+1) = 0, então A^n = 0.
>
>Cícero Thiago
>
>
>
>
>
>--
>Use o melhor sistema de busca da Internet
>Radar UOL - http://www.radaruol.com.br
>
>
>
>=
>Instruções para entrar na lista, sair da lista e usar a lista em
>http://www.mat.puc-rio.br/~nicolau/olimp/obm-l.html
>O administrador desta lista é <[EMAIL PROTECTED]>
>=
>

=
Instruções para entrar na lista, sair da lista e usar a lista em
http://www.mat.puc-rio.br/~nicolau/olimp/obm-l.html
O administrador desta lista é <[EMAIL PROTECTED]>
=



Re: [obm-l] Problema

2003-01-17 Thread bene



 

  Corrigindo,a alternativa  (b) é 400
  

  

  Nos festejos juninos, 20 casais de dançarinos são 
  colocados em círculo de tal maneira que um homem e uma mulher formando 
  um par estão situados diametralmente opostos. Durante a dança, dois 
  dançarinos adjacentes trocam de lugar enquanto todos os outros 
  permanecem na mesma posição. Essa mudança é repetida com pares 
  adjacentes até que, na posição final, os dois dançarinos de cada par 
  estejam novamente diametralmente opostos, mas na posição contrária da 
  inicial. Então o número mínimo de mudanças, de dois dançarinos 
  adjacentes, para acontecer isso é:
  (a) 20! (b) 400 (c) 10! (d) 19! (e) 
  20


Re: [obm-l] problema

2003-02-07 Thread Augusto Cesar de Oliveira Morgado
1) Por favor, quilometro se abrevia km, com k minusculo.
2) 1 no = 1,852 km/h
10 nos = 18,52 km/h
A resposta eh 370,4/18,52= 20 horas


Em Fri, 7 Feb 2003 08:19:37 -0300 (ART), elton francisco ferreira 
<[EMAIL PROTECTED]> disse:

> Se o nó é igual a uma milha marítima por hora e uma
> milha marítima é igual a 1852 m; quanto tempo uma
> embarcação com velocidade constante de 10 nós gasta
> para percorrer 370,4 Km?
> 
> ___
> Busca Yahoo!
> O melhor lugar para encontrar tudo o que você procura na Internet
> http://br.busca.yahoo.com/
> =
> Instruções para entrar na lista, sair da lista e usar a lista em
> http://www.mat.puc-rio.br/~nicolau/olimp/obm-l.html
> O administrador desta lista é <[EMAIL PROTECTED]>
> =
> 
> 

=
Instruções para entrar na lista, sair da lista e usar a lista em
http://www.mat.puc-rio.br/~nicolau/olimp/obm-l.html
O administrador desta lista é <[EMAIL PROTECTED]>
=



Re: [obm-l] problema

2003-02-07 Thread Tertuliano Carneiro
Olá!

Do enunciado, um nó equivale a 1,852 km/h. Isso é
tudo.



 --- elton francisco ferreira
<[EMAIL PROTECTED]> escreveu: > Se o nó é
igual a uma milha marítima por hora e uma
> milha marítima é igual a 1852 m; quanto tempo uma
> embarcação com velocidade constante de 10 nós gasta
> para percorrer 370,4 Km?
> 
>
___
> Busca Yahoo!
> O melhor lugar para encontrar tudo o que você
> procura na Internet
> http://br.busca.yahoo.com/
>
=
> Instruções para entrar na lista, sair da lista e
> usar a lista em
> http://www.mat.puc-rio.br/~nicolau/olimp/obm-l.html
> O administrador desta lista é
> <[EMAIL PROTECTED]>
>
= 

___
Yahoo! GeoCities
Tudo para criar o seu site: ferramentas fáceis de usar, espaço de sobra e acessórios.
http://br.geocities.yahoo.com/
=
Instruções para entrar na lista, sair da lista e usar a lista em
http://www.mat.puc-rio.br/~nicolau/olimp/obm-l.html
O administrador desta lista é <[EMAIL PROTECTED]>
=



Re: [obm-l] problema

2003-02-07 Thread Leahpar Xarm
370,4Km=370400m=200*1852=200* "nós"
dividindo o percurso pela velocidade tem-se o tempo: 20h
 elton francisco ferreira <[EMAIL PROTECTED]> wrote:
Se o nó é igual a uma milha marítima por hora e umamilha marítima é igual a 1852 m; quanto tempo umaembarcação com velocidade constante de 10 nós gastapara percorrer 370,4 Km?___Busca Yahoo!O melhor lugar para encontrar tudo o que você procura na Internethttp://br.busca.yahoo.com/=Instruções para entrar na lista, sair da lista e usar a lista emhttp://www.mat.puc-rio.br/~nicolau/olimp/obm-l.htmlO administrador desta lista é <[EMAIL PROTECTED]>=Busca Yahoo! 
O serviço de busca mais completo da Internet. O que você pensar o Yahoo! encontra.

Re: [obm-l] problema

2003-02-24 Thread Cláudio \(Prática\)


> Dos 800 sargentos formados pela EsSa a cada ano, 5%
> pedem para sair do exército ao completarem 5 anos de
> serviço, a quantidade de sargentos formados pela EsSa
> após 12 anos e que estão em atividade é?
>

Supondo que no 5o. aniversário de formatura de uma dada turma, os 5% saem e
os 760 restantes ficam até a aposentadoria (que não ocorre antes de 13
anos), então a solução é:

800 recém formados
800 formados há 1 ano
...
800 formados há 4 anos  ==> Sub-Total = 5*800 = 4.000

760 formados há 5 anos
...
760 formados há 12 anos  ==> Sub-Total = 8*760 = 6.080

Total em atividade = 10.080.

Um abraço,
Claudio.

=
Instruções para entrar na lista, sair da lista e usar a lista em
http://www.mat.puc-rio.br/~nicolau/olimp/obm-l.html
O administrador desta lista é <[EMAIL PROTECTED]>
=


Re: [obm-l] Problema

2003-03-24 Thread Johann Peter Gustav Lejeune Dirichlet
Esse e muito legalCaiu numa IMO,so nao me lembro o ano.A ideia e trabalhar um PCP com as diferenças.qualquer coisa va nas Eureka! 6,7,8,9 pra conferir a ideia.
Na verdade isso e o Teorema de Schur.
 Claudio Buffara <[EMAIL PROTECTED]> wrote:
Caros colegas da lista:Aqui vai um que esta dando trabalho:O conjunto {1,2,...,1978} eh particionado em 6 subconjuntos. Prove que umdestes subconjuntos contem um elemento que eh igual a soma de dois elementos(nao necessariamente distintos) deste mesmo subconjunto.Agradeco qualquer ajuda.Um abraco,Claudio.=Instruções para entrar na lista, sair da lista e usar a lista emhttp://www.mat.puc-rio.br/~nicolau/olimp/obm-l.htmlO administrador desta lista é <[EMAIL PROTECTED]>=Yahoo! Mail 
O melhor e-mail gratuito da internet: 6MB de espaço, antivírus, acesso POP3, filtro contra spam.

Re: [obm-l] Problema

2003-03-25 Thread Domingos Jr.
Também tenho quebrado a cabeça com ele...

Uma primeira idéia foi considerar que existe uma divisão em 6 partições onde
nenhum elemento é soma de outros dois pertencendo a mesma partição e, para
cada partição, definir um conjunto de elementos que NÃO podem entrar na
partição, pois se entrasse haveria um elemento (não necessariamente o que
estaria para entrar) que desrespeitará a propriedade desejada.

A partir daí, queria provar que a intersecção desses 6 conjuntos não seria
nunca vazia, o que indicaria que existe um inteiro que não pode ser inserido
em nenhuma partição pois quebraria a nossa regra.

Para provar que a intersecção não é vazia eu imaginei que fosse possível
somar as cardinalidades de cada conjunto e verificar que o número é
suficientemente grande para exigir que um elemento esteja em todos os 6
conjuntos.

Infelizmente, não tenho conseguido mostrar elementos (claro, distintos
dentro de um mesmo conjunto) em quantidade suficiente para realizar uma
prova como a que propus acima.

Vou continuar pensando no problema e, quem sabe, algo se ilumina!
Alguma outra idéia?

[ ]'s

> Caros colegas da lista:
>
> Aqui vai um que esta dando trabalho:
>
> O conjunto {1,2,...,1978} eh particionado em 6 subconjuntos. Prove que um
> destes subconjuntos contem um elemento que eh igual a soma de dois
elementos
> (nao necessariamente distintos) deste mesmo subconjunto.
>
> Agradeco qualquer ajuda.
>
> Um abraco,
> Claudio.

=
Instruções para entrar na lista, sair da lista e usar a lista em
http://www.mat.puc-rio.br/~nicolau/olimp/obm-l.html
O administrador desta lista é <[EMAIL PROTECTED]>
=


Re: [obm-l] Problema

2003-03-25 Thread Cláudio \(Prática\)
O prof. Morgado me mandou uma solução, a qual eu estou relutando em
consultar.

Por enquanto, eu consegui provar que se {1,2,...,1978} for particionado em 2
(ao invés de 6) subconjuntos, então um destes terá um subconjunto da forma
{a,b,a+b} ou {a,2a}.

Estou trabalhando no caso de 3 subconjuntos. Por enquanto, nada

Um abraço,
Claudio.
- Original Message -
From: "Domingos Jr." <[EMAIL PROTECTED]>
To: <[EMAIL PROTECTED]>
Sent: Tuesday, March 25, 2003 4:35 PM
Subject: Re: [obm-l] Problema


> Também tenho quebrado a cabeça com ele...
>
> Uma primeira idéia foi considerar que existe uma divisão em 6 partições
onde
> nenhum elemento é soma de outros dois pertencendo a mesma partição e, para
> cada partição, definir um conjunto de elementos que NÃO podem entrar na
> partição, pois se entrasse haveria um elemento (não necessariamente o que
> estaria para entrar) que desrespeitará a propriedade desejada.
>
> A partir daí, queria provar que a intersecção desses 6 conjuntos não seria
> nunca vazia, o que indicaria que existe um inteiro que não pode ser
inserido
> em nenhuma partição pois quebraria a nossa regra.
>
> Para provar que a intersecção não é vazia eu imaginei que fosse possível
> somar as cardinalidades de cada conjunto e verificar que o número é
> suficientemente grande para exigir que um elemento esteja em todos os 6
> conjuntos.
>
> Infelizmente, não tenho conseguido mostrar elementos (claro, distintos
> dentro de um mesmo conjunto) em quantidade suficiente para realizar uma
> prova como a que propus acima.
>
> Vou continuar pensando no problema e, quem sabe, algo se ilumina!
> Alguma outra idéia?
>
> [ ]'s
>
> > Caros colegas da lista:
> >
> > Aqui vai um que esta dando trabalho:
> >
> > O conjunto {1,2,...,1978} eh particionado em 6 subconjuntos. Prove que
um
> > destes subconjuntos contem um elemento que eh igual a soma de dois
> elementos
> > (nao necessariamente distintos) deste mesmo subconjunto.
> >
> > Agradeco qualquer ajuda.
> >
> > Um abraco,
> > Claudio.
>
> =
> Instruções para entrar na lista, sair da lista e usar a lista em
> http://www.mat.puc-rio.br/~nicolau/olimp/obm-l.html
> O administrador desta lista é <[EMAIL PROTECTED]>
> =

=
Instruções para entrar na lista, sair da lista e usar a lista em
http://www.mat.puc-rio.br/~nicolau/olimp/obm-l.html
O administrador desta lista é <[EMAIL PROTECTED]>
=


Re: [obm-l] Problema

2003-03-28 Thread Cláudio \(Prática\)
É isso aí. Parabéns.

Esse é o tipo de problema em que persistência é recompensada.

Um abraço,
Claudio.

- Original Message -
From: "Domingos Jr." <[EMAIL PROTECTED]>
To: <[EMAIL PROTECTED]>
Sent: Friday, March 28, 2003 1:49 PM
Subject: Re: [obm-l] Problema


> Acho que consegui:
>
> Vamos começar montando partições de forma a usar o menor número de
elementos
> necessários e sempre com a exigência de que nenhum elemento pode ser
> expresso como soma de outros dois (possivelmente o mesmo).
>
> Considere sempre que os elementos estão ordenados já que toda hora estarei
> trabalhando com a diferença de elementos.
>
> Pelo PCP, existe uma partição com pelo menos 330 elementos, seja ela P1 e
> {x1, x2, ..., x330} contido em P1
> o conjunto D1 = {x2 - x1, x3 - x1, ..., x330 - x1} tem elementos todos
> distintos e com certeza nenhum elemento de D1 pode ser colocado em P1.
> |D1| = 329, pelo PCP, temos que uma das partições P2, ..., P5 tem pelo
menos
> 66 elementos.
> seja I = {i1, ..., i66} contido em {2, ..., 330} índices tais que:
> { x[i1] - x1, ..., x[i66] - x1 } contido em P2
> o conjunto D2 = { (x[i2]-x1)-(x[i1]-x1), ..., (x[i66]-x1)-(x[i1]-x1) }
=
> { x[i2] - x[i1], ..., x[i66] - x[i1] } tem todos os elementos
> distintos e com certeza nenhum deles pertence a P1 ou P2 (vc entende por
> que?), |D2|=65 e, repetiremos o argumento...
> seja J = {j1, ..., j17} contido em I tal que:
> { x[j1] - x[i1], ..., x[j17] - x[i1] } contido em P3
> D3 = { x[j2] - x[j1], ..., x[j17] - x[j1] }, com nenhum elemento
em
> P1, P2 ou P3
> seja K = {k1, ..., k6} contido em J tal que:
> { x[k1] - x[j1], ..., x[k6] - x[j1] } contido em P4
> D4 = { x[k2] - x[k1], ..., x[k6] - x[k1] }, com nenhum elemento em
> P1, P2, P3 ou P4
> seja L = {l1, l2, l3} contido em K tal que:
> { x[l1] - x[k1], ..., x[l3] - x[k1] } contido em P5
> D5 = { x[l2] - x[l1], x[l3] - x[l1] }, com nenhum elemento em P1,
> P2, P3, P4 ou P5
> D5 contido em P6
> mas então, se queremos que em P6 não haja nenhum elemento que seja
a
> soma de outros dois, então x[l3] - x[l2] não pertence a P6, mas também não
> pode pertencer a nenhuma outra partição, basta ver os índices l3 e l2 como
> índices em K, J, I e {2, 330} que vemos que esse inteiro é uma diferença
> entre dois elementos de cada uma das partições!
>
> Se não tem nenhum erro, acho que matei o problema! (Ufa, eu tinha proposto
> um deadline para hj, se não conseguisse ia desistir...).
>
> [ ]'s
>
> =
> Instruções para entrar na lista, sair da lista e usar a lista em
> http://www.mat.puc-rio.br/~nicolau/olimp/obm-l.html
> O administrador desta lista é <[EMAIL PROTECTED]>
> =

=
Instruções para entrar na lista, sair da lista e usar a lista em
http://www.mat.puc-rio.br/~nicolau/olimp/obm-l.html
O administrador desta lista é <[EMAIL PROTECTED]>
=


Re: [obm-l] Problema

2003-03-28 Thread Domingos Jr.
> É isso aí. Parabéns.
>
> Esse é o tipo de problema em que persistência é recompensada.

hehehe, é mesmo... esse problema era da IMO mesmo?
a propósito, a solução do Morgado é diferente da minha?

[ ]'s

=
Instruções para entrar na lista, sair da lista e usar a lista em
http://www.mat.puc-rio.br/~nicolau/olimp/obm-l.html
O administrador desta lista é <[EMAIL PROTECTED]>
=


Re: [obm-l] Problema

2003-03-28 Thread Claudio Buffara
Nao sei se eh da IMO ou nao. Eu vi num artiga da Eureka sobre o principio
das gavetas e a solucao do Morgado era essencialmente a mesma.

Alias, eu to bem enrolado na questao dos doces de jaca e jilo da vinganca
olimpica, apesar das dicas do Nicolau. Voce chegou a dar uma olhada?
 
Um abraco,
Claudio.

on 28.03.03 15:43, Domingos Jr. at [EMAIL PROTECTED] wrote:

>> É isso aí. Parabéns.
>> 
>> Esse é o tipo de problema em que persistência é recompensada.
> 
> hehehe, é mesmo... esse problema era da IMO mesmo?
> a propósito, a solução do Morgado é diferente da minha?
> 
> [ ]'s
> 
> =
> Instruções para entrar na lista, sair da lista e usar a lista em
> http://www.mat.puc-rio.br/~nicolau/olimp/obm-l.html
> O administrador desta lista é <[EMAIL PROTECTED]>
> =
> 

=
Instruções para entrar na lista, sair da lista e usar a lista em
http://www.mat.puc-rio.br/~nicolau/olimp/obm-l.html
O administrador desta lista é <[EMAIL PROTECTED]>
=


Re: [obm-l] Problema

2003-03-28 Thread Augusto Cesar de Oliveira Morgado
Foi da IMO de 1978, sim.
Morgado


Em Sat, 29 Mar 2003 00:57:08 -0300, Claudio Buffara <[EMAIL PROTECTED]> disse:

> Nao sei se eh da IMO ou nao. Eu vi num artiga da Eureka sobre o principio
> das gavetas e a solucao do Morgado era essencialmente a mesma.
> 
> Alias, eu to bem enrolado na questao dos doces de jaca e jilo da vinganca
> olimpica, apesar das dicas do Nicolau. Voce chegou a dar uma olhada?
>  
> Um abraco,
> Claudio.
> 
> on 28.03.03 15:43, Domingos Jr. at [EMAIL PROTECTED] wrote:
> 
> >> É isso aí. Parabéns.
> >> 
> >> Esse é o tipo de problema em que persistência é recompensada.
> > 
> > hehehe, é mesmo... esse problema era da IMO mesmo?
> > a propósito, a solução do Morgado é diferente da minha?
> > 
> > [ ]'s
> > 
> > =
> > Instruções para entrar na lista, sair da lista e usar a lista em
> > http://www.mat.puc-rio.br/~nicolau/olimp/obm-l.html
> > O administrador desta lista é <[EMAIL PROTECTED]>
> > =
> > 
> 
> =
> Instruções para entrar na lista, sair da lista e usar a lista em
> http://www.mat.puc-rio.br/~nicolau/olimp/obm-l.html
> O administrador desta lista é <[EMAIL PROTECTED]>
> =
> 
> 

=
Instruções para entrar na lista, sair da lista e usar a lista em
http://www.mat.puc-rio.br/~nicolau/olimp/obm-l.html
O administrador desta lista é <[EMAIL PROTECTED]>
=


Re: [obm-l] Problema

2003-03-29 Thread Domingos Jr.
> Nao sei se eh da IMO ou nao. Eu vi num artiga da Eureka sobre o principio
> das gavetas e a solucao do Morgado era essencialmente a mesma.

Ah, o Morgado já respondeu, IMO 1978, tinha cara mesmo...

> Alias, eu to bem enrolado na questao dos doces de jaca e jilo da vinganca
> olimpica, apesar das dicas do Nicolau. Voce chegou a dar uma olhada?

Ainda não olhei direito. Infelizmente tenho uns trabalhos pra fazer, umas
matérias pra estudar e falta um pouco de tempo para ver esses problemas!

> Um abraco,
> Claudio.

[ ]'s

PS: Por que ninguém da lista apaga as partes irrelevantes da mensagem?!

=
Instruções para entrar na lista, sair da lista e usar a lista em
http://www.mat.puc-rio.br/~nicolau/olimp/obm-l.html
O administrador desta lista é <[EMAIL PROTECTED]>
=


Re: [obm-l] problema

2003-06-05 Thread Claudio Buffara



Oi, Ricardo:
 
Na verdade, isso sai por uma aplicacao direta da 
desigualdade do rearranjo.
 
Suponhamos s.p.d.g. que 0 < a(1) <= a(2) 
<= ... <= a(n).
 
Entao, 0 < 1/a(n) <= 1/a(n-1) <= ... <= 
1/a(1)
 
A desigualdade do rearranjo diz que, para qualquer 
reordenacao b(1), ..., b(n) dos a(i), vale o seguinte:
a(1)*(1/a(1)) + a(2)*(1/a(2)) + ... + a(n)*(1/a(n)) 
<= b(1)*(1/a(1)) + b(2)*(1/a(2)) + ... + b(n)*(1/a(n))
 
Ou seja:
1 + 1 +  + 1 = n <= b(1)/a(1) + b(2)/a(2) + 
... + b(n)/a(n)
 
Um abraco,
Claudio.

  - Original Message - 
  From: 
  Ricardo Prins 
  To: [EMAIL PROTECTED] 
  Sent: Thursday, June 05, 2003 9:48 
  PM
  Subject: [obm-l] problema
  
  é, morgado, não consegui. desisto. 
   
  prove que, se b(1),b(2),b(3),...,b(n) é uma 
  reordenação dos números positivos a(1),a(2),...,a(n), então
   
  b(1)/a(1) + b(2)/a(2) + ... + b(n)/a(n) >= 
  n
   
  bom, a dica foi usar desigualdade das 
  médias...tá... somatório dos a(i)/n >= raiz enésima do produtório dos 
  a(i)...mas não consigo pensar em mais nadatentei indução tb não saiu...o 
  que faço?


Re: [obm-l] problema

2003-06-05 Thread Carlos Gustavo Tamm de Araujo Moreira
   Caro Claudio,
   E' interessante notar que isso da' uma prova da desigualdade das medias
aritmetica e geometrica usando a desigualdade do rearranjo (nesse caso na
versao que determina o menor produto interno possivel de um vetor por um
rearranjo seu) : sejam x(1),...,x(n) positivos. Nao ha' perda de
generalidade em supor que seu produto e' 1 (senao dividimos todos pela sua
media geometrica). Seja a(1)=1, a(2)=x(1), a(3)=x(1).x(2), ... ,
a(n)=x(1)x(2)...x(n-1). Note que a(n)=1/x(n). Temos entao, pela desigualdade
abaixo, n<=a(1)/a(n)+a(2)/a(1)+a(3)/a(2)+...+a(n)/a(n-1)=
=x(n)+x(1)+x(2)+...+x(n-1), que e' o que queriamos provar.
   Abracos,
   Gugu

>
>Oi, Ricardo:
>
>Na verdade, isso sai por uma aplicacao direta da desigualdade do =
>rearranjo.
>
>Suponhamos s.p.d.g. que 0 < a(1) <=3D a(2) <=3D ... <=3D a(n).
>
>Entao, 0 < 1/a(n) <=3D 1/a(n-1) <=3D ... <=3D 1/a(1)
>
>A desigualdade do rearranjo diz que, para qualquer reordenacao b(1), =
>..., b(n) dos a(i), vale o seguinte:
>a(1)*(1/a(1)) + a(2)*(1/a(2)) + ... + a(n)*(1/a(n)) <=3D b(1)*(1/a(1)) + =
>b(2)*(1/a(2)) + ... + b(n)*(1/a(n))
>
>Ou seja:
>1 + 1 +  + 1 =3D n <=3D b(1)/a(1) + b(2)/a(2) + ... + b(n)/a(n)
>
>Um abraco,
>Claudio.
>  - Original Message -=20
>  From: Ricardo Prins=20
>  To: [EMAIL PROTECTED]
>  Sent: Thursday, June 05, 2003 9:48 PM
>  Subject: [obm-l] problema
>
>
>  =E9, morgado, n=E3o consegui. desisto.=20
>
>  prove que, se b(1),b(2),b(3),...,b(n) =E9 uma reordena=E7=E3o dos =
>n=FAmeros positivos a(1),a(2),...,a(n), ent=E3o
>
>  b(1)/a(1) + b(2)/a(2) + ... + b(n)/a(n) >=3D n
>
>  bom, a dica foi usar desigualdade das m=E9dias...t=E1... somat=F3rio =
>dos a(i)/n >=3D raiz en=E9sima do produt=F3rio dos a(i)...mas n=E3o =
>consigo pensar em mais nadatentei indu=E7=E3o tb n=E3o saiu...o que =
>fa=E7o?
=
Instruções para entrar na lista, sair da lista e usar a lista em
http://www.mat.puc-rio.br/~nicolau/olimp/obm-l.html
=


Re: [obm-l] problema

2003-06-06 Thread Ricardo Prins
Muito obrigado a todos vocês! Ontem eu acabei achando na Eureka 5 muitas
coisas interessantes quanto à desigualdades em geral...preciso estudar mais!
aqui vai outro problema que eu não consegui resolver...é do lidski...

uma sucessão infinita de números x1,x2,x3,...,xn,... (x1<>0) para qualquer
n>=3 satisfaz à condição
(x1²+x2²++xn-1²)(x2²+x3³+...+xn²)=(x1x2+x2x3+...+xn-1xn)²

demonstrar que x1,x2,x3,...,xn,... são termos sucessivos de uma p.g.
- Original Message -
From: "Carlos Gustavo Tamm de Araujo Moreira" <[EMAIL PROTECTED]>
To: <[EMAIL PROTECTED]>
Sent: Friday, June 06, 2003 1:27 AM
Subject: Re: [obm-l] problema


>Caro Claudio,
>E' interessante notar que isso da' uma prova da desigualdade das medias
> aritmetica e geometrica usando a desigualdade do rearranjo (nesse caso na
> versao que determina o menor produto interno possivel de um vetor por um
> rearranjo seu) : sejam x(1),...,x(n) positivos. Nao ha' perda de
> generalidade em supor que seu produto e' 1 (senao dividimos todos pela sua
> media geometrica). Seja a(1)=1, a(2)=x(1), a(3)=x(1).x(2), ... ,
> a(n)=x(1)x(2)...x(n-1). Note que a(n)=1/x(n). Temos entao, pela
desigualdade
> abaixo, n<=a(1)/a(n)+a(2)/a(1)+a(3)/a(2)+...+a(n)/a(n-1)=
> =x(n)+x(1)+x(2)+...+x(n-1), que e' o que queriamos provar.
>Abracos,
>Gugu
>
> >
> >Oi, Ricardo:
> >
> >Na verdade, isso sai por uma aplicacao direta da desigualdade do =
> >rearranjo.
> >
> >Suponhamos s.p.d.g. que 0 < a(1) <=3D a(2) <=3D ... <=3D a(n).
> >
> >Entao, 0 < 1/a(n) <=3D 1/a(n-1) <=3D ... <=3D 1/a(1)
> >
> >A desigualdade do rearranjo diz que, para qualquer reordenacao b(1), =
> >..., b(n) dos a(i), vale o seguinte:
> >a(1)*(1/a(1)) + a(2)*(1/a(2)) + ... + a(n)*(1/a(n)) <=3D b(1)*(1/a(1)) +
=
> >b(2)*(1/a(2)) + ... + b(n)*(1/a(n))
> >
> >Ou seja:
> >1 + 1 +  + 1 =3D n <=3D b(1)/a(1) + b(2)/a(2) + ... + b(n)/a(n)
> >
> >Um abraco,
> >Claudio.
> >  - Original Message -=20
> >  From: Ricardo Prins=20
> >  To: [EMAIL PROTECTED]
> >  Sent: Thursday, June 05, 2003 9:48 PM
> >  Subject: [obm-l] problema
> >
> >
> >  =E9, morgado, n=E3o consegui. desisto.=20
> >
> >  prove que, se b(1),b(2),b(3),...,b(n) =E9 uma reordena=E7=E3o dos =
> >n=FAmeros positivos a(1),a(2),...,a(n), ent=E3o
> >
> >  b(1)/a(1) + b(2)/a(2) + ... + b(n)/a(n) >=3D n
> >
> >  bom, a dica foi usar desigualdade das m=E9dias...t=E1... somat=F3rio =
> >dos a(i)/n >=3D raiz en=E9sima do produt=F3rio dos a(i)...mas n=E3o =
> >consigo pensar em mais nadatentei indu=E7=E3o tb n=E3o saiu...o que =
> >fa=E7o?
> =
> Instruções para entrar na lista, sair da lista e usar a lista em
> http://www.mat.puc-rio.br/~nicolau/olimp/obm-l.html
> =
>
=
Instruções para entrar na lista, sair da lista e usar a lista em
http://www.mat.puc-rio.br/~nicolau/olimp/obm-l.html
=


Re: [obm-l] problema

2003-06-06 Thread Cláudio \(Prática\)
Oi, Gugu:

Eu já tinha visto essa dedução de MG <= MA a partir do rearranjo mas, apesar
de interessante, é "mágica" demais pro meu gosto.

Eu prefiro aquela em que você vai trocando Xmin e Xmax por G e Xmin*Xmax/G
até que todos os números fiquem iguais.

Um abraço,
Claudio.


- Original Message -
From: "Carlos Gustavo Tamm de Araujo Moreira" <[EMAIL PROTECTED]>
To: <[EMAIL PROTECTED]>
Sent: Friday, June 06, 2003 1:27 AM
Subject: Re: [obm-l] problema


>Caro Claudio,
>E' interessante notar que isso da' uma prova da desigualdade das medias
> aritmetica e geometrica usando a desigualdade do rearranjo (nesse caso na
> versao que determina o menor produto interno possivel de um vetor por um
> rearranjo seu) : sejam x(1),...,x(n) positivos. Nao ha' perda de
> generalidade em supor que seu produto e' 1 (senao dividimos todos pela sua
> media geometrica). Seja a(1)=1, a(2)=x(1), a(3)=x(1).x(2), ... ,
> a(n)=x(1)x(2)...x(n-1). Note que a(n)=1/x(n). Temos entao, pela
desigualdade
> abaixo, n<=a(1)/a(n)+a(2)/a(1)+a(3)/a(2)+...+a(n)/a(n-1)=
> =x(n)+x(1)+x(2)+...+x(n-1), que e' o que queriamos provar.
>Abracos,
>Gugu
>

=
Instruções para entrar na lista, sair da lista e usar a lista em
http://www.mat.puc-rio.br/~nicolau/olimp/obm-l.html
=


Re: [obm-l] problema

2003-06-06 Thread Cláudio \(Prática\)

- Original Message -
From: "Ricardo Prins" <[EMAIL PROTECTED]>
To: <[EMAIL PROTECTED]>
Sent: Friday, June 06, 2003 11:45 AM
Subject: Re: [obm-l] problema


> Muito obrigado a todos vocês! Ontem eu acabei achando na Eureka 5 muitas
> coisas interessantes quanto à desigualdades em geral...preciso estudar
mais!
> aqui vai outro problema que eu não consegui resolver...é do lidski...
>
> uma sucessão infinita de números x1,x2,x3,...,xn,... (x1<>0) para qualquer
> n>=3 satisfaz à condição
> (x1²+x2²++xn-1²)(x2²+x3³+...+xn²)=(x1x2+x2x3+...+xn-1xn)²
>
> demonstrar que x1,x2,x3,...,xn,... são termos sucessivos de uma p.g.


Oi, Ricardo:

Aqui temos que provar que existe um no. real q tal que:
x(2) = q*x(1),  x(3) = q*x(2),  ..., x(n) = q*x(n-1),  ...

Uma idéia é usar a desigualdade de Cauchy-Schwarz relativa ao produto
escalar de 2 vetores do R^m.

Ela diz o seguinte:
Se x e y são vetores do R^m, então | x . y |^2 <= || x ||^2 * || y ||^2, com
igualdade sss x e y são LI
onde:
x . y = produto escalar de x e y
| x . y | = módulo do número real  x . y
|| x || = módulo do vetor x = raiz(x(1)^2 + x(2)^2 + ... + x(m)^2)

A expressão do enunciado é justamente | x . y |^2 = || x ||^2 * || y ||^2
com:
x = ( x(1) , x(2) , ... , x(n-1) ) e y = ( x(2) , x(3) , ... , x(n) ):
vetores do R^(n-1).

Logo, concluímos que x e y são LI  ==>
existe um no. real q tal que y = qx  ==>
x(2) = q*x(1), x(3) = q*x(2), ..., x(n) = q*x(n-1) ==>
os x(k) são termos de uma PG.

Esse artigo da Eureka 5 tem uma demonstração de Cauchy-Schwarz (lá é chamada
apensa de Cauchy - é a proposição no. 3).

Um abraço,
Claudio.

=
Instruções para entrar na lista, sair da lista e usar a lista em
http://www.mat.puc-rio.br/~nicolau/olimp/obm-l.html
=


Re: [obm-l] problema

2003-06-06 Thread Ricardo Prins
Obrigado mais uma vez, Cláudio!

R.Prins
- Original Message -
From: "Cláudio (Prática)" <[EMAIL PROTECTED]>
To: <[EMAIL PROTECTED]>
Sent: Friday, June 06, 2003 1:04 PM
Subject: Re: [obm-l] problema


>
> - Original Message -
> From: "Ricardo Prins" <[EMAIL PROTECTED]>
> To: <[EMAIL PROTECTED]>
> Sent: Friday, June 06, 2003 11:45 AM
> Subject: Re: [obm-l] problema
>
>
> > Muito obrigado a todos vocês! Ontem eu acabei achando na Eureka 5 muitas
> > coisas interessantes quanto à desigualdades em geral...preciso estudar
> mais!
> > aqui vai outro problema que eu não consegui resolver...é do lidski...
> >
> > uma sucessão infinita de números x1,x2,x3,...,xn,... (x1<>0) para
qualquer
> > n>=3 satisfaz à condição
> > (x1²+x2²++xn-1²)(x2²+x3³+...+xn²)=(x1x2+x2x3+...+xn-1xn)²
> >
> > demonstrar que x1,x2,x3,...,xn,... são termos sucessivos de uma p.g.
>
>
> Oi, Ricardo:
>
> Aqui temos que provar que existe um no. real q tal que:
> x(2) = q*x(1),  x(3) = q*x(2),  ..., x(n) = q*x(n-1),  ...
>
> Uma idéia é usar a desigualdade de Cauchy-Schwarz relativa ao produto
> escalar de 2 vetores do R^m.
>
> Ela diz o seguinte:
> Se x e y são vetores do R^m, então | x . y |^2 <= || x ||^2 * || y ||^2,
com
> igualdade sss x e y são LI
> onde:
> x . y = produto escalar de x e y
> | x . y | = módulo do número real  x . y
> || x || = módulo do vetor x = raiz(x(1)^2 + x(2)^2 + ... + x(m)^2)
>
> A expressão do enunciado é justamente | x . y |^2 = || x ||^2 * || y ||^2
> com:
> x = ( x(1) , x(2) , ... , x(n-1) ) e y = ( x(2) , x(3) , ... , x(n) ):
> vetores do R^(n-1).
>
> Logo, concluímos que x e y são LI  ==>
> existe um no. real q tal que y = qx  ==>
> x(2) = q*x(1), x(3) = q*x(2), ..., x(n) = q*x(n-1) ==>
> os x(k) são termos de uma PG.
>
> Esse artigo da Eureka 5 tem uma demonstração de Cauchy-Schwarz (lá é
chamada
> apensa de Cauchy - é a proposição no. 3).
>
> Um abraço,
> Claudio.
>
> =
> Instruções para entrar na lista, sair da lista e usar a lista em
> http://www.mat.puc-rio.br/~nicolau/olimp/obm-l.html
> =
>
=
Instruções para entrar na lista, sair da lista e usar a lista em
http://www.mat.puc-rio.br/~nicolau/olimp/obm-l.html
=


Re: [obm-l] Problema

2003-05-29 Thread Augusto Cesar de Oliveira Morgado
Suponho que sejam axiomas de corpo, ou pelo menos de dominio de integridade.

ab = ac implica ab - ac = 0 implica a(b-c) = 0 implica a=0 ou b-c=0  implica a=0 ou 
b=c que, se a diferente de 0, implica b=c


Em Thu, 1 Jan 1998 04:20:38 -0200, Fernando <[EMAIL PROTECTED]> disse:

> 
> Gostaria de ajuda para o seguinte problema:
>  1) Mostrar que,se no sistema de axiomas substituirmos a propriedade cancelativa da 
> Multiplicação( a(diferente de 0) , se ab=ac, então b=c) pela proposição 
> (a.b=0.Então, a=0 ou b=0),  a propriedade cancelativa da multiplicação pode ser 
> demonstrada a partir de novo sistema de axiomas. 
> 
> Atenciosamente, 
> Fernando. 
=
Instruções para entrar na lista, sair da lista e usar a lista em
http://www.mat.puc-rio.br/~nicolau/olimp/obm-l.html
=


RE: [obm-l] Problema

2003-05-31 Thread Leandro Lacorte Recôva









Acho que isso e uma
definicao de modulo  Definicao nao se discute, aceita  

 

Mod(x) = x se x>=0 

     -x se x <
0. 

 

Definicao !!! 

 

Se mod(x)=0, entao pela
definicao, voce mostra que x=0. 

 

 

 

-Original Message-
From:
[EMAIL PROTECTED] [mailto:[EMAIL PROTECTED] On Behalf Of Fernando
Sent: Wednesday, December 31, 1997
6:21 PM
To: [EMAIL PROTECTED]
Subject: [obm-l] Problema

 



Gostaria de ajudar o para o seguinte problema:





Mostrar que:





/x/ >/ = 0 e /x/=0 se e somente se x= 0





Atenciosamente,





Fernando.










Re: [obm-l] Problema

2003-05-31 Thread Cláudio \(Prática\)



Oi, Fernando:
 
Por definição de valor absoluto:
|x| = x, se  x > 0
|x| = -x, se x < 0
|0| = 0.
 
Assim:
x > 0 ==> |x| = x > 0
x < 0 ==> |x| = -x > 0
x = 0 ==> |x| = |0| = 0
 
Ou seja, |x| >= 0, e é igual a zero se e somente 
se x = 0.
 
Você está estudando alguma coisa relacionada a 
fundamentos da matemática?
 
Um abraço,
Claudio.
 

  - Original Message - 
  From: 
  Fernando 
  To: [EMAIL PROTECTED] 
  Sent: Wednesday, December 31, 1997 11:21 
  PM
  Subject: [obm-l] Problema
  
  Gostaria de ajudar o para o seguinte problema:
  Mostrar que:
  /x/ >/ = 0 e /x/=0 se e somente se x= 0
  Atenciosamente,
  Fernando.


Re: [[obm-l] Problema]

2003-05-31 Thread Artur Costa Steiner
Oi Fernando,
temos que /x/ =x se x>=0
e /x/ = -x se x<0 (supondo-se que x eh um numero real)
Se x>0, entao /x/ = x >0
Se x= 0 entao /x/= x =0
E x <0 entao /x/ = -x >0
Logo, se /x/ =0, entao x=0. Esta eh a unica possibilidade

Por outro lado, se x=0, entao /x/ = x =0. Disso concluimos que /x/=0 se e
somente se x =0.

Se /x/ >/ = 0 significar /x/ <= 0, entao pelo que vimos, x=0, pois em qualquer
outro caso temos /x/ >0 (nunca temos /x/ <0). E se x =0, entao /x/ =0 <=0, o
que completa a prova.
Artur

> - 
> Gostaria de ajudar o para o seguinte problema:
> Mostrar que:
> /x/ >/ = 0 e /x/=0 se e somente se x= 0
> Atenciosamente,
> Fernando.
> 



=
Instruções para entrar na lista, sair da lista e usar a lista em
http://www.mat.puc-rio.br/~nicolau/olimp/obm-l.html
=


Re: [obm-l] Problema

2003-06-02 Thread Carlos Maçaranduba
Olhe eu sei quem ele é e não tive nenhuma intenção de
ofende-lo.Confesso que fiquei até surpreso ao notar a
reação de algumas para UM SIMPLES E-MAIL.Aposto que
ele possui coisas mais importantes para se preocupar.
Vamos encerrar este assunto por aqui e como conselho
,acho que eu voce e todos desta lista deveriam ignorar
dessas bobagens.


 --- Frederico Reis Marques de Brito
<[EMAIL PROTECTED]> escreveu: > Não quero fazer
patrulha ideológica, mas a sucessão
> de emails sobre esse 
> tema me causou profunda estranheza. Sou obrigado a
> dizer que além de não 
> saber o que vem a ser um polinômio, o Maçaranduba tb
> não deve ter noção de 
> quem é o professor Morgado. É evidente que todos
> somos passíveis de erros, 
> mas para se afirmar que o MOrgado deu um
> contra-exemplo "furado" é 
> necessário pensar um bocado e ter muita coragem(
> acho que eu não teria )  
> sobre isto antes, sobretudo em se tratando de um
> assunto tão simples, como o 
> que motivou todas essas mensagens.
> 
> Enfim, acho que alguns partici´pantes da lista
> deveriam sintonizar-se.
> 
> Frederico.
> 
> 
> >From: "A. C. Morgado" <[EMAIL PROTECTED]>
> >Reply-To: [EMAIL PROTECTED]
> >To: [EMAIL PROTECTED]
> >Subject: Re: [obm-l] Problema de aneis de
> polinomios
> >Date: Sat, 31 May 2003 20:27:03 -0300
> >
> >Voce sabe o que eh um polinomio? Isso que voce esta
> indicando aih eu nao 
> >sei o que eh, mas polinomio nao eh. Imagine se o
> seu corpo for R: 
> >produtorio de (x-w), w percorrendo os reais.
> >Voce sabe o que eh um polinomio nulo? Voce ja se
> deu ao trabalho de olhar a 
> >resposta do Carlos Cesar a sua pergunta?
> >Esta eh minha ultima manifestaçao a respeito.
> >
> >Carlos Maçaranduba wrote:
> >
> >>usando o seu argumento eu poderia dizer que no
> caso de
> >>um corpo infinito , eu poderia construir um
> produtorio
> >>de (x - w) infinitos para todo w que pertence ao
> >>corpo.Isto é possivel pelo teorema das raizes de
> um
> >>polinomio num corpo.Entao eu obteria um polinomio
> não
> >>nulo de infinitos fatoresNão é um polinomio
> nulo
> >>como (x - 1)(x - 0)mod2 do seu exemplo..
> >>
> >>--- "A. C. Morgado" <[EMAIL PROTECTED]>
> >>escreveu: > Um polinomio eh  nulo quando os seus
> >>coeficientes
> >>
> >>
> >>>sao nulos, o que nao eh o caso do polinomio f(x)
> = x^2 + x no corpo dos
> >>>inteiros modulo 2; dois dos coeficientes desse
> polinomio sao iguais a
> >>>1.
> >>>Entao, eh falso que f = 0.
> >>>Mas f(0) = f(1) = 0, ou seja, f(w) = 0 para todo
> w
> >>>em Z2.
> >>>
> >>>Leia a resposta que lhe foi mandada por Carlos
> >>>César de Araújo e convença-se de que isso eh um
> contra-exemplo sim..
> >>>
> >>>Carlos Maçaranduba wrote:
> >>>
> >>>
> >>>
> >>>>Pode ser que seja problema de interpretação, mas
> eu
> >>>>acho que isto não é contra-exemplo PORQUE:
> >>>>
> >>>>->Pela hipotese, eu nao deveria considerar,
> mesmo
> >>>>
> >>>>
> >>>para
> >>>
> >>>
> >>>>um corpo finito que f(w) = 0 ,para todo elemento
> w
> >>>>pertencente ao corpo finito e  CONCLUIR QUE f =0
> É
> >>>>FALSO NESTE CASO
> >>>>
> >>>>->UM CONTRA-EXEMPLO BOM NAO SERIA RESPEITANDO O
> QUE
> >>>>
> >>>>
> >>>EU
> >>>
> >>>
> >>>>DISSE ACIMA
> >>>>
> >>>>
> >>>>
> >>>>
> >>>>--- "A. C. Morgado" <[EMAIL PROTECTED]>
> >>>>escreveu: > f(x) = x^2 + x  em Z(2) eh um
> >>>>contraexemplo.
> >>>>
> >>>>
> >>>>
> >>>>
> >>>>>Carlos Maçaranduba wrote:
> >>>>>
> >>>>>
> >>>>>
> >>>>>
> >>>>>
> >>>>>>Seja k um corpo infinito.Se f pertence a k[x]
> é
> >>>>>>
> >>>>>>
> >>>tal
> >>>
> >>>
> >>>>>>que f(w) = 0 para todo elemento w pertencente
> a
> >>>>>>
> >>>>>>
> >>>k,
> >>>
> >>&

Re: [obm-l] Problema

2003-06-02 Thread + BRiSSiU +


Do que vocês estão falando?
Tratem esses assuntos por fora da lista!


- Original Message -
From: Carlos Maçaranduba <[EMAIL PROTECTED]>
To: <[EMAIL PROTECTED]>
Sent: Sunday, June 01, 2003 5:20 PM
Subject: Re: [obm-l] Problema


> Olhe eu sei quem ele é e não tive nenhuma intenção de
> ofende-lo.Confesso que fiquei até surpreso ao notar a
> reação de algumas para UM SIMPLES E-MAIL.Aposto que
> ele possui coisas mais importantes para se preocupar.
> Vamos encerrar este assunto por aqui e como conselho
> ,acho que eu voce e todos desta lista deveriam ignorar
> dessas bobagens.
>
>
>  --- Frederico Reis Marques de Brito
> <[EMAIL PROTECTED]> escreveu: > Não quero fazer
> patrulha ideológica, mas a sucessão
> > de emails sobre esse
> > tema me causou profunda estranheza. Sou obrigado a
> > dizer que além de não
> > saber o que vem a ser um polinômio, o Maçaranduba tb
> > não deve ter noção de
> > quem é o professor Morgado. É evidente que todos
> > somos passíveis de erros,
> > mas para se afirmar que o MOrgado deu um
> > contra-exemplo "furado" é
> > necessário pensar um bocado e ter muita coragem(
> > acho que eu não teria )
> > sobre isto antes, sobretudo em se tratando de um
> > assunto tão simples, como o
> > que motivou todas essas mensagens.
> >
> > Enfim, acho que alguns partici´pantes da lista
> > deveriam sintonizar-se.
> >
> > Frederico.
> >
> >
> > >From: "A. C. Morgado" <[EMAIL PROTECTED]>
> > >Reply-To: [EMAIL PROTECTED]
> > >To: [EMAIL PROTECTED]
> > >Subject: Re: [obm-l] Problema de aneis de
> > polinomios
> > >Date: Sat, 31 May 2003 20:27:03 -0300
> > >
> > >Voce sabe o que eh um polinomio? Isso que voce esta
> > indicando aih eu nao
> > >sei o que eh, mas polinomio nao eh. Imagine se o
> > seu corpo for R:
> > >produtorio de (x-w), w percorrendo os reais.
> > >Voce sabe o que eh um polinomio nulo? Voce ja se
> > deu ao trabalho de olhar a
> > >resposta do Carlos Cesar a sua pergunta?
> > >Esta eh minha ultima manifestaçao a respeito.
> > >
> > >Carlos Maçaranduba wrote:
> > >
> > >>usando o seu argumento eu poderia dizer que no
> > caso de
> > >>um corpo infinito , eu poderia construir um
> > produtorio
> > >>de (x - w) infinitos para todo w que pertence ao
> > >>corpo.Isto é possivel pelo teorema das raizes de
> > um
> > >>polinomio num corpo.Entao eu obteria um polinomio
> > não
> > >>nulo de infinitos fatoresNão é um polinomio
> > nulo
> > >>como (x - 1)(x - 0)mod2 do seu exemplo..
> > >>
> > >>--- "A. C. Morgado" <[EMAIL PROTECTED]>
> > >>escreveu: > Um polinomio eh  nulo quando os seus
> > >>coeficientes
> > >>
> > >>
> > >>>sao nulos, o que nao eh o caso do polinomio f(x)
> > = x^2 + x no corpo dos
> > >>>inteiros modulo 2; dois dos coeficientes desse
> > polinomio sao iguais a
> > >>>1.
> > >>>Entao, eh falso que f = 0.
> > >>>Mas f(0) = f(1) = 0, ou seja, f(w) = 0 para todo
> > w
> > >>>em Z2.
> > >>>
> > >>>Leia a resposta que lhe foi mandada por Carlos
> > >>>César de Araújo e convença-se de que isso eh um
> > contra-exemplo sim..
> > >>>
> > >>>Carlos Maçaranduba wrote:
> > >>>
> > >>>
> > >>>
> > >>>>Pode ser que seja problema de interpretação, mas
> > eu
> > >>>>acho que isto não é contra-exemplo PORQUE:
> > >>>>
> > >>>>->Pela hipotese, eu nao deveria considerar,
> > mesmo
> > >>>>
> > >>>>
> > >>>para
> > >>>
> > >>>
> > >>>>um corpo finito que f(w) = 0 ,para todo elemento
> > w
> > >>>>pertencente ao corpo finito e  CONCLUIR QUE f =0
> > É
> > >>>>FALSO NESTE CASO
> > >>>>
> > >>>>->UM CONTRA-EXEMPLO BOM NAO SERIA RESPEITANDO O
> > QUE
> > >>>>
> > >>>>
> > >>>EU
> > >>>
> > >>>
> > >>>>DISSE ACIMA
> > >>>>
> > >>>>
> > >>>>
> > >>>>
> > >>>>--- "A. C. Morgado" <[EMAIL PROTEC

RE: [obm-l] Problema

2003-06-03 Thread Artur Costa Steiner
Oi Fernando,
Temos que (n!)^2 = 1 2...(n-1)n X 1 2...(n-1)n. = 1.2...(n-1).n X n
(n-1)...2 1= 1n 2(n-1)...n.1. Temos entao que (n!)^2 = Produto( k=1,n)
k(n-k+1) = Produto(k=1,n) -k^2+(n+1)k
Cada termo do produto eh portanto um polinomio do segundo grau em k, que
apresenta um maximo para k= (n+1)/2 (embora este valor nao seja inteiro
quando n eh par, se o substituirmos no polinomio obtemos um limite superior
para o mesmo) e eh portanto simetrico com relacao aa vertical k = (n+1)/2.
Logo, o polinomio, para k=1,2...n, apresenta valor minimo quando k estah nos
pontos extremos de seu dominio discreto {1,2...n}. Isto ocorre para k=1 ou
k=n e obtemos o minimo de n. Concluimos assim que cada termo do produto eh
>=n e que, portanto, (n!)^2 >= Produto(k=1,n) n = n^n. Para n=1,2 obtemos
igualdade e para n>=3 desigualdade estrita, pois diversos termos do produto
tornam-se maiores que n. Por exemplo, para k =2 obtemos 2(n-1)>n para n>=3.
Isto prova a desigualdade.
Deve haver tambem uma outra demonstracao baseada em inducao finita. 
Um abraco
Artur  

-Original Message-
From: [EMAIL PROTECTED]
[mailto:[EMAIL PROTECTED] On Behalf Of Fernando
Sent: Thursday, January 01, 1998 12:56 AM
To: [EMAIL PROTECTED]
Subject: [obm-l] Problema




Gostaria de ajudar o para o seguinte problema:
Mostrar que:
se o inteiro n>/ 3, então ( n!)^2 > n^n
Atenciosamente,
Fernando.
<>

Re: [obm-l] Problema

2003-06-03 Thread Cláudio \(Prática\)



Gostaria de ajudar o para o seguinte problema:
Mostrar que:
se o inteiro n>/ 3, então ( n!)^2 > n^n
Atenciosamente,
Fernando.
 
Oi, Fernando:
 
Vou dar só uma dica: use o seguinte:
 
(1)  (n!)^2 = [1*2*...*n]*[n*(n-1)*...*1] = 
[1*n]*[2*(n-1)]*...*[n*1]
 
(2) Para n >= 3 e 
1 <= k <= n, k*(n+1-k) >= n, com igualdade se e somente se k = 1 ou k = 
n. 
Dem:
k*(n+1-k) - n = -(k^2 - (n+1)*k + n) = 
-(k - n)*(k - 1) = (n - k)*(k - 1) >= 0 para 1 <= k <= n.
Igualdade se e somente se n - k = 0  ou  
k - 1 = 0.
 
Um abraço,
Claudio.


Re: [obm-l] Problema

2003-03-28 Thread Domingos Jr.
Acho que consegui:

Vamos começar montando partições de forma a usar o menor número de elementos
necessários e sempre com a exigência de que nenhum elemento pode ser
expresso como soma de outros dois (possivelmente o mesmo).

Considere sempre que os elementos estão ordenados já que toda hora estarei
trabalhando com a diferença de elementos.

Pelo PCP, existe uma partição com pelo menos 330 elementos, seja ela P1 e
{x1, x2, ..., x330} contido em P1
o conjunto D1 = {x2 - x1, x3 - x1, ..., x330 - x1} tem elementos todos
distintos e com certeza nenhum elemento de D1 pode ser colocado em P1.
|D1| = 329, pelo PCP, temos que uma das partições P2, ..., P5 tem pelo menos
66 elementos.
seja I = {i1, ..., i66} contido em {2, ..., 330} índices tais que:
{ x[i1] - x1, ..., x[i66] - x1 } contido em P2
o conjunto D2 = { (x[i2]-x1)-(x[i1]-x1), ..., (x[i66]-x1)-(x[i1]-x1) } =
{ x[i2] - x[i1], ..., x[i66] - x[i1] } tem todos os elementos
distintos e com certeza nenhum deles pertence a P1 ou P2 (vc entende por
que?), |D2|=65 e, repetiremos o argumento...
seja J = {j1, ..., j17} contido em I tal que:
{ x[j1] - x[i1], ..., x[j17] - x[i1] } contido em P3
D3 = { x[j2] - x[j1], ..., x[j17] - x[j1] }, com nenhum elemento em
P1, P2 ou P3
seja K = {k1, ..., k6} contido em J tal que:
{ x[k1] - x[j1], ..., x[k6] - x[j1] } contido em P4
D4 = { x[k2] - x[k1], ..., x[k6] - x[k1] }, com nenhum elemento em
P1, P2, P3 ou P4
seja L = {l1, l2, l3} contido em K tal que:
{ x[l1] - x[k1], ..., x[l3] - x[k1] } contido em P5
D5 = { x[l2] - x[l1], x[l3] - x[l1] }, com nenhum elemento em P1,
P2, P3, P4 ou P5
D5 contido em P6
mas então, se queremos que em P6 não haja nenhum elemento que seja a
soma de outros dois, então x[l3] - x[l2] não pertence a P6, mas também não
pode pertencer a nenhuma outra partição, basta ver os índices l3 e l2 como
índices em K, J, I e {2, 330} que vemos que esse inteiro é uma diferença
entre dois elementos de cada uma das partições!

Se não tem nenhum erro, acho que matei o problema! (Ufa, eu tinha proposto
um deadline para hj, se não conseguisse ia desistir...).

[ ]'s

=
Instruções para entrar na lista, sair da lista e usar a lista em
http://www.mat.puc-rio.br/~nicolau/olimp/obm-l.html
O administrador desta lista é <[EMAIL PROTECTED]>
=


Re: [obm-l] Problema

2003-06-17 Thread guilherme S.
Fernando <[EMAIL PROTECTED]> wrote:




 
 
Gostaria de ajudar o para o seguinte problema:
Mostrar que:
se o inteiro n>/ 3, então ( n!)^2 > n^n
Atenciosamente,
Fernando.
 
 
 
o primeiro membro da desigualdade eequivalente a:
 
[ n(n-1)(n-2)..1]^2=n*1*(n-1)*2..(n-k+1)k1*n
 
sabemos que :n-k+1>0 ->k
 
entaooproblemaemostrar que:(n-k+1)k>n
hipotese: (n-k+1)k<=n ->k^2-k(n+1)+n>=0  -> k<=ou k>=n,que e um absurdo,de (1), logoa hipotesee falsa e (n-k+1)>n
 
esefazendo oprodutorio n vezes com k variando de 1 a n obtemos que: (n!)^2>n^n cqd.Yahoo! Mail 
Mais espaço, mais segurança e gratuito: caixa postal de 6MB, antivírus, proteção contra spam.

Re: [obm-l] Problema

2003-07-31 Thread Domingos Jr.
Em uma reunião há 201 pessoas de 5 nacionalidades
diferentes. Sabe-se que, em cada grupo de 6, ao menos 2
tem a mesma idade.

Demonstrar que ha ao menos 5 pessoas do mesmo pais, da
mesma idade e do mesmo sexo.

--- x ---

Seja X a idade de uma das pessoas da reunião
Isole nosso amigo da reunião e particione as 200 pessoas restantes em grupos
de 5.
Evidentemente se colocarmos nosso amigo em cada um dos grupos haverá um
outro indivíduo com idade X.
Sendo assim garantimos que há pelo menos 41 pessoas com a idade X.
Considerando que só há dois sexos... temos pelo menos 21 pessoas do mesmo
sexo e idade X (PCP).
Como há 5 nacionalidades, deve haver uma (PCP novamente) nacionalidade que 5
pessoas com o mesmo sexo e idade X compartilham.

[ ]'s

=
Instruções para entrar na lista, sair da lista e usar a lista em
http://www.mat.puc-rio.br/~nicolau/olimp/obm-l.html
=


Re: [obm-l] Problema

2003-08-01 Thread Faelccmm
Soh nao entendi a ultima parte:
Como há 5 nacionalidades...5 pessoas com o mesmo sexo e idade X compartilham.
Nos outros casos (como 41 pessoas com a idade X e o 21 pessoas do mesmo
sexo e idade X) eu entendi perfeitamente como foi aplicado o PCP, mas nao sei como aplicar o PCP para concluir que 5 pessoas com o mesmo sexo e idade X compartilham uma nacionalidade. Por que 5 ?

Em uma mensagem de 1/8/2003 01:08:03 Hora padrão leste da Am. Sul, [EMAIL PROTECTED] escreveu:


Em uma reunião há 201 pessoas de 5 nacionalidades
diferentes. Sabe-se que, em cada grupo de 6, ao menos 2
tem a mesma idade.

Demonstrar que ha ao menos 5 pessoas do mesmo pais, da
mesma idade e do mesmo sexo.

--- x ---

Seja X a idade de uma das pessoas da reunião
Isole nosso amigo da reunião e particione as 200 pessoas restantes em grupos
de 5.
Evidentemente se colocarmos nosso amigo em cada um dos grupos haverá um
outro indivíduo com idade X.
Sendo assim garantimos que há pelo menos 41 pessoas com a idade X.
Considerando que só há dois sexos... temos pelo menos 21 pessoas do mesmo
sexo e idade X (PCP).
Como há 5 nacionalidades, deve haver uma (PCP novamente) nacionalidade que 5
pessoas com o mesmo sexo e idade X compartilham.




Re: [obm-l] Problema

2003-08-02 Thread Rafael Ando
Se existem 21 pessoas e 5 nacionalidades, pelo menos 1 nacionalidade tem 5 
pessoas (i.e., se nenhuma nacionalidade tivesse 5 pessoas entao teriamos no 
maximo 4+4+4+4+4 = 20 pessoas)

From: [EMAIL PROTECTED]
Reply-To: [EMAIL PROTECTED]
To: [EMAIL PROTECTED]
Subject: Re: [obm-l] Problema
Date: Fri, 1 Aug 2003 16:42:36 EDT
Soh nao entendi a ultima parte:
Como há 5 nacionalidades...5 pessoas com o mesmo sexo e idade X 
compartilham.
Nos outros casos (como 41 pessoas com a idade X e o 21 pessoas do mesmo
sexo e idade X) eu entendi perfeitamente como foi aplicado o PCP, mas nao 
sei
como aplicar o PCP para concluir que 5 pessoas com o mesmo sexo e idade X
compartilham uma nacionalidade. Por que 5 ?

Em uma mensagem de 1/8/2003 01:08:03 Hora padrão leste da Am. Sul,
[EMAIL PROTECTED] escreveu:
> Em uma reunião há 201 pessoas de 5 nacionalidades
> diferentes. Sabe-se que, em cada grupo de 6, ao menos 2
> tem a mesma idade.
>
> Demonstrar que ha ao menos 5 pessoas do mesmo pais, da
> mesma idade e do mesmo sexo.
>
> --- x ---
>
> Seja X a idade de uma das pessoas da reunião
> Isole nosso amigo da reunião e particione as 200 pessoas restantes em 
grupos
> de 5.
> Evidentemente se colocarmos nosso amigo em cada um dos grupos haverá um
> outro indivíduo com idade X.
> Sendo assim garantimos que há pelo menos 41 pessoas com a idade X.
> Considerando que só há dois sexos... temos pelo menos 21 pessoas do 
mesmo
> sexo e idade X (PCP).
> Como há 5 nacionalidades, deve haver uma (PCP novamente) nacionalidade 
que 5
> pessoas com o mesmo sexo e idade X compartilham.


_
MSN Messenger: instale grátis e converse com seus amigos. 
http://messenger.msn.com.br

=
Instruções para entrar na lista, sair da lista e usar a lista em
http://www.mat.puc-rio.br/~nicolau/olimp/obm-l.html
=


Re: [obm-l] Problema

2003-08-02 Thread Alexandre Terezan
Não entendi o raciocínio abaixo:

"Seja X a idade de uma das pessoas da reunião
Isole nosso amigo da reunião e particione as 200 pessoas restantes em
grupos
de 5.
Evidentemente se colocarmos nosso amigo em cada um dos grupos haverá um
outro indivíduo com idade X."

Já que num destes grupos de 5 já podiam existir 2 ou mais pessoas com a
idade Y, o que satisfaria a condição do enunciado, sem q houvesse a
necessidade de 41 pessoas com a idade X.

O raciocínio correto, a meu ver, começaria pela conclusão de que, se eu
posso garantir que em cada grupo de 6, 2 possuem a mesma idade, então
existem no máximo 5 "idades" diferentes entre as 201 pessoas.

O resto fica evidente pelo PCP.



- Original Message -
From: "Rafael Ando" <[EMAIL PROTECTED]>
To: <[EMAIL PROTECTED]>
Sent: Saturday, August 02, 2003 10:57 AM
Subject: Re: [obm-l] Problema


>
> Se existem 21 pessoas e 5 nacionalidades, pelo menos 1 nacionalidade tem 5
> pessoas (i.e., se nenhuma nacionalidade tivesse 5 pessoas entao teriamos
no
> maximo 4+4+4+4+4 = 20 pessoas)
>
> >From: [EMAIL PROTECTED]
> >Reply-To: [EMAIL PROTECTED]
> >To: [EMAIL PROTECTED]
> >Subject: Re: [obm-l] Problema
> >Date: Fri, 1 Aug 2003 16:42:36 EDT
> >
> >Soh nao entendi a ultima parte:
> >Como há 5 nacionalidades...5 pessoas com o mesmo sexo e idade X
> >compartilham.
> >Nos outros casos (como 41 pessoas com a idade X e o 21 pessoas do mesmo
> >sexo e idade X) eu entendi perfeitamente como foi aplicado o PCP, mas nao
> >sei
> >como aplicar o PCP para concluir que 5 pessoas com o mesmo sexo e idade X
> >compartilham uma nacionalidade. Por que 5 ?
> >
> >Em uma mensagem de 1/8/2003 01:08:03 Hora padrão leste da Am. Sul,
> >[EMAIL PROTECTED] escreveu:
> >
> >
> > > Em uma reunião há 201 pessoas de 5 nacionalidades
> > > diferentes. Sabe-se que, em cada grupo de 6, ao menos 2
> > > tem a mesma idade.
> > >
> > > Demonstrar que ha ao menos 5 pessoas do mesmo pais, da
> > > mesma idade e do mesmo sexo.
> > >
> > > --- x ---
> > >
> > > Seja X a idade de uma das pessoas da reunião
> > > Isole nosso amigo da reunião e particione as 200 pessoas restantes em
> >grupos
> > > de 5.
> > > Evidentemente se colocarmos nosso amigo em cada um dos grupos haverá
um
> > > outro indivíduo com idade X.
> > > Sendo assim garantimos que há pelo menos 41 pessoas com a idade X.
> > > Considerando que só há dois sexos... temos pelo menos 21 pessoas do
> >mesmo
> > > sexo e idade X (PCP).
> > > Como há 5 nacionalidades, deve haver uma (PCP novamente) nacionalidade
> >que 5
> > > pessoas com o mesmo sexo e idade X compartilham.
> >
> >
>
> _
> MSN Messenger: instale grátis e converse com seus amigos.
> http://messenger.msn.com.br
>
> =
> Instruções para entrar na lista, sair da lista e usar a lista em
> http://www.mat.puc-rio.br/~nicolau/olimp/obm-l.html
> =
>

=
Instruções para entrar na lista, sair da lista e usar a lista em
http://www.mat.puc-rio.br/~nicolau/olimp/obm-l.html
=


Re: [obm-l] Problema

2003-08-02 Thread Domingos Jr.

Não entendi o raciocínio abaixo:

"Seja X a idade de uma das pessoas da reunião
Isole nosso amigo da reunião e particione as 200 pessoas restantes em
grupos
de 5.
Evidentemente se colocarmos nosso amigo em cada um dos grupos haverá um
outro indivíduo com idade X."

Já que num destes grupos de 5 já podiam existir 2 ou mais pessoas com a
idade Y, o que satisfaria a condição do enunciado, sem q houvesse a
necessidade de 41 pessoas com a idade X.

O raciocínio correto, a meu ver, começaria pela conclusão de que, se eu
posso garantir que em cada grupo de 6, 2 possuem a mesma idade, então
existem no máximo 5 "idades" diferentes entre as 201 pessoas.

O resto fica evidente pelo PCP.

 --- x ---

Eu usei implicitamente esse fato, deveria ter sido mais claro:
"Seja X a idade de uma das pessoas da reunião
Isole nosso amigo da reunião e particione as 200 pessoas restantes em
grupos de 5 CUJAS IDADES SÃO DISTINTAS!"

Na verdade essa foi uma falha ao passar a sol. do papel para o pc...

[ ]'s

=
Instruções para entrar na lista, sair da lista e usar a lista em
http://www.mat.puc-rio.br/~nicolau/olimp/obm-l.html
=


Re: [obm-l] Problema

2003-08-02 Thread Domingos Jr.
Favor desconsiderar... ficou pior ainda!


--- x --

Eu usei implicitamente esse fato, deveria ter sido mais claro:
"Seja X a idade de uma das pessoas da reunião
Isole nosso amigo da reunião e particione as 200 pessoas restantes em
grupos de 5 CUJAS IDADES SÃO DISTINTAS!"

Na verdade essa foi uma falha ao passar a sol. do papel para o pc...

[ ]'s


=
Instruções para entrar na lista, sair da lista e usar a lista em
http://www.mat.puc-rio.br/~nicolau/olimp/obm-l.html
=


Re:[obm-l] Problema

2003-08-03 Thread claudio.buffara

 




De:
[EMAIL PROTECTED]




Para:
[EMAIL PROTECTED]




Cópia:





Data:
Thu, 31 Jul 2003 11:35:25 -0300




Assunto:
201 pessoas de 5 nacionalidades




 
 
> 
> Oi Pessoal, como posso resolver este problema ?
> 
> Obrigado e um abraço.
> 
> Amurpe.
> 
> 
> Em uma reunião há 201 pessoas de 5 nacionalidades 
> diferentes. Sabe-se que, em cada grupo de 6, ao menos 2 
> tem a mesma idade. 
> 
> Demonstrar que ha ao menos 5 pessoas do mesmo pais, da 
> mesma idade e do mesmo sexo.
> 
>
 
Oi, Amurpe:
 
Existem 5*2 = 10 combinacoes distintas de nacionalidade e sexo. 
Se no maximo 20 pessoas pertencessem a cada combinacao, o numero total de pessoas seria 20*10 = 200 < 201 ==> contradicao.
Logo, deve existir uma combinacao de nacionalidade e sexo a qual pertencem pelo menos 21 pessoas.
 
Suponhamos que, dentre estas 21 pessoas, no maximo 4 tenham a mesma idade.
 
Chamemos as pessoas de P1, P2, ..., P21, onde i < j ==> idade(Pi) <= idade(Pj).
 
Pela nossa suposicao (no maximo 4 com a mesma idade), teremos:
idade(P1) < idade(P5) < idade(P9) < idade(P13) < idade(P17) < idade(P21).
(por exemplo, se idade(P1) = idade(P5), entao P1, P2, P3, P4 e P5 teriam a mesma idade, contrariamente a nossa suposicao).
 
Ou seja, o grupo de 6 pessoas {P1,P5,P9,P13,P17,P21} nao tem duas pessoas com a mesma idade ==> contradicao.
Logo, devem haver 5 pessoas (dentre as 21) todas com a mesma idade.
 
Como as 21 pessoas tem a mesma nacionalidade e o mesmo sexo, concluimos que as 5 pessoas dentre elas tem a mesma nacionalidade, sexo e idade.
 
 
Um abraco,
Claudio.
 
 

Re: [obm-l] Problema

2003-08-04 Thread Domingos Jr.



1002 ~ -1001 (mod 2003)
1003 ~ -1000 (mod 2003)
...
2002 ~ -1 (mod 2003)
 
logo 
1002*1003*...*2002 = (-1)(-2)...(-1001) = 
-(1*2*...*1001) mod (2003)
dessa forma temos
1*2*3*...*1001 + 1002*1003*...*2002 
= 1*2*3*...*1001 - 1*2*3*...*1001 = 0 (mod 2003)
 
[ ]'s

  - Original Message - 
  From: 
  Benedito 
  To: [EMAIL PROTECTED] 
  Sent: Monday, August 04, 2003 10:56 
  AM
  Subject: [obm-l] Problema
  
  Problema
  Mostre que o abaixo é divisível por 
  2003:
  1*2*3*...*1001 + 1002*1003*...*2002.
   
  Benedito 
Freire


Re: [obm-l] Problema

2002-02-04 Thread Nicolau C. Saldanha

On Sat, Feb 02, 2002 at 08:28:34AM -0200, Jose Paulo Carneiro wrote:
> Uma professora me mencionou um jogo que eh jogado em alguns lugares do
> Rio (por exemplo, no Colegio Militar). Vou descrever: O jogo eh jogado por 4
> pessoas (apos a descricao, eh facil ver que tambem pode ser jogado por 3 ou 2
> pessoas) em uma mesa retangular, onde faca sentido falar na "vertical", que
> varia com cada jogador.  Um tabuleiro de 8 x 8 casas (naturalmente, pode-se
> pensar em n x n) eh colocado com as arestas paralelas as da mesa.  No
> tabuleiro, colocam-se ao acaso numeros inteiros em todas as casas, exceto
> em uma, onde se coloca uma estrela.  Comecando agora de um primeiro jogador e
> prosseguindo (digamos) em sentido horario, cada jogador retira (e guarda) um
> numero situado na mesma vertical (em relacao a este jogador) em que estah a
> estrela, e coloca a estrela no lugar do numero retirado.  O jogo acaba quando
> nao restam mais numeros no tabuleiro, e ganha aquele que tiver guardada a
> maior soma dos numeros retirados.  A consulta que faco, em nome desta
> professora, eh: este jogo tem estrategia vencedora?  JP

Interessante. Claramente depende dos inteiros inicialmente no tabuleiro.
O fato do jogo envolver mais de dois jogadores é sempre um complicador
pois podem ocorrer coalizões temporárias entre jogadores. Mas principalmente
eu não entendi uma coisa: pode ocorrer de um jogador na sua vez não ter
nenhum número para tirar (sem que o jogo tenha terminado ainda). E aí?
O jogador passa a vez? Ou perde, ou...? E se o jogador *puder* jogar
mas achar que todas as jogadas dele são más, ele tem o direito de passar a vez?

[]s, N.
=
Instruções para entrar na lista, sair da lista e usar a lista em
http://www.mat.puc-rio.br/~nicolau/olimp/obm-l.html
O administrador desta lista é <[EMAIL PROTECTED]>
=



Re: [obm-l] Problema

2002-02-04 Thread Jose Paulo Carneiro

Boas perguntas.
Vou repassar para a professora.
JP

- Original Message -
From: Nicolau C. Saldanha <[EMAIL PROTECTED]>
To: <[EMAIL PROTECTED]>
Sent: Monday, February 04, 2002 9:23 AM
Subject: Re: [obm-l] Problema


On Sat, Feb 02, 2002 at 08:28:34AM -0200, Jose Paulo Carneiro wrote:
> Uma professora me mencionou um jogo que eh jogado em alguns lugares do
> Rio (por exemplo, no Colegio Militar). Vou descrever: O jogo eh jogado por
4
> pessoas (apos a descricao, eh facil ver que tambem pode ser jogado por 3
ou 2
> pessoas) em uma mesa retangular, onde faca sentido falar na "vertical",
que
> varia com cada jogador.  Um tabuleiro de 8 x 8 casas (naturalmente,
pode-se
> pensar em n x n) eh colocado com as arestas paralelas as da mesa.  No
> tabuleiro, colocam-se ao acaso numeros inteiros em todas as casas, exceto
> em uma, onde se coloca uma estrela.  Comecando agora de um primeiro
jogador e
> prosseguindo (digamos) em sentido horario, cada jogador retira (e guarda)
um
> numero situado na mesma vertical (em relacao a este jogador) em que estah
a
> estrela, e coloca a estrela no lugar do numero retirado.  O jogo acaba
quando
> nao restam mais numeros no tabuleiro, e ganha aquele que tiver guardada a
> maior soma dos numeros retirados.  A consulta que faco, em nome desta
> professora, eh: este jogo tem estrategia vencedora?  JP

Interessante. Claramente depende dos inteiros inicialmente no tabuleiro.
O fato do jogo envolver mais de dois jogadores é sempre um complicador
pois podem ocorrer coalizões temporárias entre jogadores. Mas principalmente
eu não entendi uma coisa: pode ocorrer de um jogador na sua vez não ter
nenhum número para tirar (sem que o jogo tenha terminado ainda). E aí?
O jogador passa a vez? Ou perde, ou...? E se o jogador *puder* jogar
mas achar que todas as jogadas dele são más, ele tem o direito de passar a
vez?

[]s, N.
=
Instruções para entrar na lista, sair da lista e usar a lista em
http://www.mat.puc-rio.br/~nicolau/olimp/obm-l.html
O administrador desta lista é <[EMAIL PROTECTED]>
=


=
Instruções para entrar na lista, sair da lista e usar a lista em
http://www.mat.puc-rio.br/~nicolau/olimp/obm-l.html
O administrador desta lista é <[EMAIL PROTECTED]>
=



Re: [obm-l] problema

2002-02-14 Thread Eduardo Wagner

Oi Fred:

Considerando 0 < b < a < 1, a relacao eh
(a^2)(c^2 + d^2) < (1 - a^2)(a^2 - b^2).
Vou conferir as contas e em seguida mando
a solucao completa. Nesse meio tempo, outros
membros da lista devem se manifestar.

Abraco,

Wagner.

--
>From: Carlos Frederico Borges Palmeira <[EMAIL PROTECTED]>
>To: [EMAIL PROTECTED]
>Subject: [obm-l] problema
>Date: Thu, Feb 14, 2002, 17:05
>

> um problema aparentemente simples: considere uma elipse de semi-eixos a
> ,b, centrada em um ponto (c,d) do plano. Determinar condicoes em a,b,c,d
> para que a elipse esteja toda contida no interior do disco x^2+y^2 <1.
>
> Fred palmeira
>
> =
> InstruÁžes para entrar na lista, sair da lista e usar a lista em
> http://www.mat.puc-rio.br/~nicolau/olimp/obm-l.html
> O administrador desta lista È <[EMAIL PROTECTED]>
> =
=
Instruções para entrar na lista, sair da lista e usar a lista em
http://www.mat.puc-rio.br/~nicolau/olimp/obm-l.html
O administrador desta lista é <[EMAIL PROTECTED]>
=



Re: [obm-l] Problema

2002-03-06 Thread Eduardo Wagner

Tem algum erro. Se AA' = CC' o triangulo eh isosceles.

--
>From: André <[EMAIL PROTECTED]>
>To: [EMAIL PROTECTED]
>Subject: [obm-l] Problema
>Date: Wed, Mar 6, 2002, 8:02
>

> Saudacoes,
>
> Alguem pode me ajudar na solucao do problema abaixo?
>
> Dado um triangulo ABC, seja AA' e CC' as bissetrizes dos angulos angA e angC
> respectivamente. Sabendo que angA > angB > angC e AA' = CC', prove que:
>
> c*sen((angA-angB)/2)=a*sen((angB-angC)/2).
>
> Agradeco...
>
> Andre/RJ.
>
> _
> Oi! Você quer um iG-mail gratuito?
> Então clique aqui: http://registro.ig.com.br/censo/igmail
>
> =
> Instruções para entrar na lista, sair da lista e usar a lista em
> http://www.mat.puc-rio.br/~nicolau/olimp/obm-l.html
> O administrador desta lista é <[EMAIL PROTECTED]>
> =
=
Instruções para entrar na lista, sair da lista e usar a lista em
http://www.mat.puc-rio.br/~nicolau/olimp/obm-l.html
O administrador desta lista é <[EMAIL PROTECTED]>
=



Re: [obm-l] Problema.

2002-03-09 Thread Antonio Jose Gonzales Alves

Seja 
G a quantidade inicial de grama que havia no pasto,
c a quantidade de grama que uma cabra come por dia e
d a quantidade de grama que cresce por dia.

Assim, após 20 dias a quantidade de grama no pasto passou a ser G +
20d. Como essa grama foi comida por 50 cabras em 20 dias, temos que:
G + 20d = 20*50*c (I)
Analogamente, temos G + 30d = 30*40*c (II)

De (I) e (II), temos d = G/30 e c = G/600.

Agora seja x a quantidade de cabras necessárias para comer um pasto
inteiro em 40 dias. Temos que a quantidade de grama a ser comida é 
G + 40*G/30.

Logo G + 40*G/30 = 40*x*G/600  <-> x = 35 cabras.

Um abraço a todos.
 |=-=-=-=-=-=-=-=-=-=-=-=-=-=-=-|
 |--- \\|// |
 |--- (o o) |
 |--- oOOo~(_)~oOOo |
 |--|
 |- Toninho <:\ |
 |www.linux.ime.usp.br/~gonzales|
 |--|
 |=-=-=-=-=-=-=-=-=-=-=-=-=-=-=-|

On Sat, 9 Mar 2002, Moacyr Moreira wrote:

> Bom dia,
> 
>  Se alguém puder me ajudar a resolver este
> problema, eu ficaria muito grato.
> 
>  Problema: A Sra. Barnabé, proprietária de um sítio em
> Arceburgo, notou que o capim crescia no pasto todo com
> igual rapidez e espessura. Observando o consumo de
> capim pela criação de cabras, a Sra. Barnabé notou que
> 50 cabras consumiam um pasto inteiro em 20 dias e que
> 40 cabras levaram 30 dias para consumir um pasto
> inteiro. A partir desses dados, a Sra. Barnabé quer
> determinar qual o número de cabras necessárias para
> consumir um pasto inteiro em 40 dias. Resolva o
> problema.
> 
>  Abraço,
> 
> Moacyr.
> 
> 
>___
> Yahoo! Empregos
> O trabalho dos seus sonhos pode estar aqui. Cadastre-se hoje mesmo no Yahoo! 
>Empregos e tenha acesso a milhares de vagas abertas!
> http://br.empregos.yahoo.com/
> =
> Instruções para entrar na lista, sair da lista e usar a lista em
> http://www.mat.puc-rio.br/~nicolau/olimp/obm-l.html
> O administrador desta lista é <[EMAIL PROTECTED]>
> =
> 

=
Instruções para entrar na lista, sair da lista e usar a lista em
http://www.mat.puc-rio.br/~nicolau/olimp/obm-l.html
O administrador desta lista é <[EMAIL PROTECTED]>
=



Re: [obm-l] problema

2002-03-17 Thread Alexandre F. Terezan

Dúvida: Os números que eu obtiver podem comecar com o algarismo 0?
-Mensagem Original-
De: <[EMAIL PROTECTED]>
Para: <[EMAIL PROTECTED]>
Enviada em: Domingo, 17 de Março de 2002 20:33 Terezan
Assunto: [obm-l] problema


Olá amigos da lista , se possível me ajudem com esse problema abaixo , desde
ja , grato.
Rick

Suprima cem díditos do número 1234567891011121314151617...5960 de modo a
obter o menor número possível . A seguir , refaça o mesmo para obter o maior
número possível . A soma dos algarismos desses dois números é:




--
Use o melhor sistema de busca da Internet
Radar UOL - http://www.radaruol.com.br



=
Instruções para entrar na lista, sair da lista e usar a lista em
http://www.mat.puc-rio.br/~nicolau/olimp/obm-l.html
O administrador desta lista é <[EMAIL PROTECTED]>
=


=
Instruções para entrar na lista, sair da lista e usar a lista em
http://www.mat.puc-rio.br/~nicolau/olimp/obm-l.html
O administrador desta lista é <[EMAIL PROTECTED]>
=



Re: [obm-l] Problema.

2002-04-21 Thread Marcos Aurélio Almeida da Silva

bom cada cabra come "c m2" de capim, k cabras comerão "kc m2" de capim por
dia,
o capim cresce "r m2" por dia,

na primeira situação:

em 20 dias, o pasto cresceu 20*r, mas as cabras comeram 20*k*c, como não
sobrou nada:

p+20*r = 20*k*50(eq 1)
k = (p+20*r) / 20*50 = (p+20r)/1000  (eq 2)

na segunda situação

p+30*r = 30*k*40
k = (p+30*r) / 30*40 = (p+30r)/1200  (eq 3)

k=k (eq 2 = eq 3)
(p+20r) / 1000 = (p+30r)/1200
(p+20r) / 5 = (p+30r) / 6
6p + 120r = 5p + 150r

p  = 30r

logo, na terceira :

k = (p+40*r) / 40*c
   = (30r+40r) / 40c
   = 70r/40c
   = 7/4  (r/c)

p+20r = 20*k*50(usando eq 1)
30r+20r = 20 * (7/4) * r/c * 50
1 = (7/4) * (20 / c)
c = 35 cabras  ???

- Original Message -
From: "Moacyr Moreira" <[EMAIL PROTECTED]>
To: <[EMAIL PROTECTED]>
Sent: Saturday, March 09, 2002 9:55 AM
Subject: [obm-l] Problema.


> Bom dia,
>
>  Se alguém puder me ajudar a resolver este
> problema, eu ficaria muito grato.
>
>  Problema: A Sra. Barnabé, proprietária de um sítio em
> Arceburgo, notou que o capim crescia no pasto todo com
> igual rapidez e espessura. Observando o consumo de
> capim pela criação de cabras, a Sra. Barnabé notou que
> 50 cabras consumiam um pasto inteiro em 20 dias e que
> 40 cabras levaram 30 dias para consumir um pasto
> inteiro. A partir desses dados, a Sra. Barnabé quer
> determinar qual o número de cabras necessárias para
> consumir um pasto inteiro em 40 dias. Resolva o
> problema.
>
>  Abraço,
>
> Moacyr.
>
>

___
> Yahoo! Empregos
> O trabalho dos seus sonhos pode estar aqui. Cadastre-se hoje mesmo no
Yahoo! Empregos e tenha acesso a milhares de vagas abertas!
> http://br.empregos.yahoo.com/
> =
> Instruções para entrar na lista, sair da lista e usar a lista em
> http://www.mat.puc-rio.br/~nicolau/olimp/obm-l.html
> O administrador desta lista é <[EMAIL PROTECTED]>
> =




_
Do You Yahoo!?
Get your free @yahoo.com address at http://mail.yahoo.com

=
Instruções para entrar na lista, sair da lista e usar a lista em
http://www.mat.puc-rio.br/~nicolau/olimp/obm-l.html
O administrador desta lista é <[EMAIL PROTECTED]>
=



Re: [obm-l] Problema

2002-05-22 Thread ezer


Dica: O que eh exatamente sqtr(x)? A que deducao este fato
pode nos levar? Pense mais um pouco..

Esse problema eh semelhante a um que jah foi discutido aqui..,
soh que este eh verificavel :c)

Ha uma resolucao do problema no final do email, mas tente antes
vc mesmo ;c)

Abracos,

Ezer F. da Silva



On 21 May 2002 at 23:22, Eduardo Quintas wrote:

> resolva a equação :
> 
> x^(sqrt x) = 1/2
> 
> PS.: x elevado a raiz quadrada de x = 1/2
> 
> =
> Instruções para entrar na lista, sair da lista e usar a lista em
> http://www.mat.puc-rio.br/~nicolau/olimp/obm-l.html
> O administrador desta lista é <[EMAIL PROTECTED]>
> =



x^sqrt(x) = 1/2  = x^x^(1/2) = 1/2

Como 1/2 eh igual a x^x^(1/2), podemos substituir na equacao:
x^x^x^x^(1/2) = 1/2. E podemos substituir infinitamente.
x^x^x^x^x^... = 1/2. 

Como 1/2 = x^x^x^x^x^..., entao podemos substituir
e ficaremos com x^(1/2) = 1/2  (isso soh eh possivel pq infinito - 1 = infinito, ou 
seja,
um conjunto infinito nao aumenta nem diminui quando lhe tiramos ou
acrescentamos um numero finito de termos)

x^(1/2) = 1/2
x = 1/4

Substituindo para verificar:
x^sqtr(x) = 1/2  |  (1/4)^sqtr(1/4) = 1/2 | sqtr(1/4) = 1/2 | 1/2 = 1/2
=
Instruções para entrar na lista, sair da lista e usar a lista em
http://www.mat.puc-rio.br/~nicolau/olimp/obm-l.html
O administrador desta lista é <[EMAIL PROTECTED]>
=



Re: [obm-l] Problema

2004-05-29 Thread Lista OBM
Meu caro Cláudio, meu nome é Éder Franklin da Silva. Meu login é Lista OBM porque tenho mais de um e-mail no Yahoo (pra ser mais exato possuo 4 e-mail´s) e porque essa lista envia muitas mensagens por dia. Daí preferi criar um e-mail especialmentre para ela e, nada mais natural colocar um login (Lista OBM ) que se indentique com o nome dela. Agora não entendi porque você quer saber meu nome. 
Ah, obrigado pela solução. Acho que está correta.
 
Grato, Éder Franklin da Silva. 
 
Está bom assim?"claudio.buffara" <[EMAIL PROTECTED]> wrote:

From: Lista OBM <[EMAIL PROTECTED]> To: [EMAIL PROTECTED] Sent: Fri, 28 May 2004 11:41:34 -0300 (ART) Subject: [obm-l] Problema > Gostaria de saber se alguém poderia me ajudar com o seguinte problema: > Sejam A e B anéis ordenados. Diz-se que um homomorfismo injetivo f: A --> B preserva ordem se, para todo a > 0 em A, tivermos f(a) >; 0. Sejam K um corpo ordenado e f: Q --> K um homomorfismo injetivo dos números racionais em K. Mostre que, necessariamente, f preserva a ordem. >   > Grato desde já com a possível ajuda de vocês. >
Antes de mais nada, qual o seu nome? Espero sinceramente que nao seja "Lista OBM"...
 
Agora, sobre o problema:
 
Como f eh injetivo, K contem uma copia isomorfica de Q. Alem disso, eh facil ver que f(1) = 1_k = elemento neutro da multiplicacao em K, e que isso implica que se m/n pertence a Q (m, n inteiros), entao f(m/n) = m_k/n_k, onde:
m_k = 1_k + 1_k + ... + 1_k (m parcelas).
 
Como K eh ordenado, 1_k = 1_k*1_k > 0_k, ou seja, f(1) > 0_k ==> f(1) eh positivo em K.
Logo, f(m) = f(1+1+...+1) = 1_k + 1_k + ... + 1_k = m_k tambem eh positivo em K.
 
1_k = f(1) = f(m*1/m) = f(m)*f(1/m) = m_k*f(1/m) ==>
f(1/m) = 1_k/m_k.
Logo, m > 0 ==> 1/m > 0 ==> f(1/m) = 1/m_k > 0_k.
 
Assim, provamos que se m > 0 e n > 0 em Q, entao f(m) > 0_k e f(1/n) > 0_k.
 
Agora, dado um racional positivo m/n (m,n inteiros), podemos assumir s.p.d.g. que m > 0 e n > 0 e, portanto, f(m/n) = f(m*1/n) = f(m)*f(1/n) > 0_k.
 
Ou seja, se a > 0 em Q, entao f(a) > 0_k em K.
 
[]s,
Claudio.
 
 
 
 
 
 
 
 Yahoo! Messenger - Fale com seus amigos online. Instale agora!

Re: [obm-l] Problema

2004-05-29 Thread claudio.buffara

Oi, Eder:
 
Na verdade, eu soh queria saber do que chama-lo. Alias, um apelido tambem serviria, pois aqui temos um Dirichlet, um Qwert Smith, e uma P. da Silva, por exemplo. Eu soh acho que seria meio esquisito comecar uma msg com "Caro Lista OBM", mas se voce preferir assim, nao vejo problema algum.
 
[]s,
Claudio.
 
 




De:
[EMAIL PROTECTED]




Para:
[EMAIL PROTECTED]




Cópia:





Data:
Sat, 29 May 2004 17:20:48 -0300 (ART)




Assunto:
Re: [obm-l] Problema




 
 
> Meu caro Cláudio, meu nome é Éder Franklin da Silva. Meu login é Lista OBM porque tenho mais de um e-mail no Yahoo (pra ser mais exato possuo 4 e-mail´s) e porque essa lista envia muitas mensagens por dia. Daí preferi criar um e-mail especialmentre para ela e, nada mais natural colocar um login (Lista OBM ) que se indentique com o nome dela. Agora não entendi porque você quer saber meu nome. 
> Ah, obrigado pela solução. Acho que está correta.
>  
> Grato, Éder Franklin da Silva. 
>  
> Está bom assim?

Re: [obm-l] Problema

2004-05-29 Thread Lista OBM
Não precisava pedir desculpas, isso acontece com qualquer um.Augusto Cesar de Oliveira Morgado <[EMAIL PROTECTED]> wrote:

Claro, que burrice a minha! A soma é preservada, mas a multiplicação não! Desculpe. Morgado == Mensagem  enviada  pelo  CIP  WebMAIL  - Nova Geração - v. 2.1 CentroIn Internet Provider          http://www.centroin.com.br Tel: (21) 2542-4849, (21) 2295-3331        Fax: (21) 2295-2978 Empresa 100% Brasileira - Desde 1992 prestando servicos online -- Original Message --- From: Lista OBM <[EMAIL PROTECTED]> To: [EMAIL PROTECTED] Sent: Sat, 29 May 2004 08:54:55 -0300 (ART) Subject: Re: [obm-l] Problema > Meu caro Morgado, > não sei se estou equivocado, mas a aplicação que você definiu não é um homomorfismo, pois: f(x.y) =
 -(x.y) = -x.y e f(x).f(y) =(-x).(-y) = x.y, ou seja, f(x.y) é diferente de f(x).f(y). Além do mais, num homomorfismo f entre domínios de integridade sempre temos que: ou f leva o elemento identidade (em relção a multiplicação) do domínio no elemento identidade do contadomínio ou f é a função constante zero. De fato, f(0) = f(0+0) = f(0)+f(0) =>f(0) = 0 e f(1) = f(1.1) = f(1).f(1) => f(1)[1 - f(1)] = 0 => 1 = f(1) ou f(1) = 0. Se f(1) = 0 então segue que f(x) = f(x.1) = f(x).f(1) = f(x).0 = 0, para todo x do domínio, ou seja, f é a função constate zero. Assim, nunca pode ocorrer f(1) = -1 num homomorfismo entre corpos.    > > Augusto Cesar de Oliveira Morgado <[EMAIL PROTECTED]> wrote: 
Isso é falso! Tome K=Q e defina f por f(x)=-x. > 1 é positivo e f(1) não é. > > == > Mensagem  enviada  pelo  CIP  WebMAIL  - Nova Geração - v. 2.1 > CentroIn Internet Provider          http://www.centroin.com.br > Tel: (21) 2542-4849, (21) 2295-3331        Fax: (21) 2295-2978 > Empresa 100% Brasileira - Desde 1992 prestando servicos online > > -- Original Message --- > From: Lista OBM <[EMAIL PROTECTED]> > To: [EMAIL PROTECTED] > Sent: Fri, 28 May 2004 11:41:34 -0300 (ART) > Subject: [obm-l] Problema > > > Gostaria de saber se alguém poderia me
 ajudar com o seguinte problema: > > Sejam A e B anéis ordenados. Diz-se que um homomorfismo injetivo f: A --> B preserva ordem se, para todo a > 0 em A, tivermos f(a) >; 0. Sejam K um corpo ordenado e f: Q --> K um homomorfismo injetivo dos números racionais em K. Mostre que, necessariamente, f preserva a ordem. > >   > > Grato desde já com a possível ajuda de vocês. > > > > 

Yahoo! Messenger - Fale com seus amigos online. Instale agora! > --- End of Original Message --- > > > 

Yahoo! Messenger - Fale com seus amigos online. Instale agora! --- End of Original Message --- Yahoo! Messenger - Fale com seus amigos online. Instale agora!

Re: [obm-l] Problema

2004-05-29 Thread Lista OBM
Meu caro Cláudio, desculpe-me se fui groço contigo. Mas p0ode me chamar de Éder mesmo."claudio.buffara" <[EMAIL PROTECTED]> wrote:


Oi, Eder:
 
Na verdade, eu soh queria saber do que chama-lo. Alias, um apelido tambem serviria, pois aqui temos um Dirichlet, um Qwert Smith, e uma P. da Silva, por exemplo. Eu soh acho que seria meio esquisito comecar uma msg com "Caro Lista OBM", mas se voce preferir assim, nao vejo problema algum.
 
[]s,
Claudio.
 
 




De:
[EMAIL PROTECTED]




Para:
[EMAIL PROTECTED]




Cópia:





Data:
Sat, 29 May 2004 17:20:48 -0300 (ART)




Assunto:
Re: [obm-l] Problema




 
 
> Meu caro Cláudio, meu nome é Éder Franklin da Silva. Meu login é Lista OBM porque tenho mais de um e-mail no Yahoo (pra ser mais exato possuo 4 e-mail´s) e porque essa lista envia muitas mensagens por dia. Daí preferi criar um e-mail especialmentre para ela e, nada mais natural colocar um login (Lista OBM ) que se indentique com o nome dela. Agora não entendi porque você quer saber meu nome. 
> Ah, obrigado pela solução. Acho que está correta.
>  
> Grato, Éder Franklin da Silva. 
>  
> Está bom assim?Yahoo! Messenger - Fale com seus amigos online. Instale agora!

Re: [obm-l] Problema

2004-05-30 Thread Johann Peter Gustav Lejeune Dirichlet
Bem, essa pergunta eu tambem iria fazer.
E que e meio estranho voce ver uma mensagem da lista para a lista...
Que tal voce colocar um pseudonimo, algo como Eder na Lista OBM?
Isto nao gasta nada (alem de alguns caracteres :) )
Te mais!!!
 
Lista OBM <[EMAIL PROTECTED]> wrote:

Meu caro Cláudio, meu nome é Éder Franklin da Silva. Meu login é Lista OBM porque tenho mais de um e-mail no Yahoo (pra ser mais exato possuo 4 e-mail´s) e porque essa lista envia muitas mensagens por dia. Daí preferi criar um e-mail especialmentre para ela e, nada mais natural colocar um login (Lista OBM ) que se indentique com o nome dela. Agora não entendi porque você quer saber meu nome. 
Ah, obrigado pela solução. Acho que está correta.
 
Grato, Éder Franklin da Silva. 
 
Está bom assim?"claudio.buffara" <[EMAIL PROTECTED]> wrote:

From: Lista OBM <[EMAIL PROTECTED]> To: [EMAIL PROTECTED] Sent: Fri, 28 May 2004 11:41:34 -0300 (ART) Subject: [obm-l] Problema > Gostaria de saber se alguém poderia me ajudar com o seguinte problema: > Sejam A e B anéis ordenados. Diz-se que um homomorfismo injetivo f: A --> B preserva ordem se, para todo a > 0 em A, tivermos f(a) >; 0. Sejam K um corpo ordenado e f: Q --> K um homomorfismo injetivo dos números racionais em K. Mostre que, necessariamente, f preserva a ordem. >   > Grato desde já com a possível ajuda de vocês. >
Antes de mais nada, qual o seu nome? Espero sinceramente que nao seja "Lista OBM"...
 
Agora, sobre o problema:
 
Como f eh injetivo, K contem uma copia isomorfica de Q. Alem disso, eh facil ver que f(1) = 1_k = elemento neutro da multiplicacao em K, e que isso implica que se m/n pertence a Q (m, n inteiros), entao f(m/n) = m_k/n_k, onde:
m_k = 1_k + 1_k + ... + 1_k (m parcelas).
 
Como K eh ordenado, 1_k = 1_k*1_k > 0_k, ou seja, f(1) > 0_k ==> f(1) eh positivo em K.
Logo, f(m) = f(1+1+...+1) = 1_k + 1_k + ... + 1_k = m_k tambem eh positivo em K.
 
1_k = f(1) = f(m*1/m) = f(m)*f(1/m) = m_k*f(1/m) ==>
f(1/m) = 1_k/m_k.
Logo, m > 0 ==> 1/m > 0 ==> f(1/m) = 1/m_k > 0_k.
 
Assim, provamos que se m > 0 e n > 0 em Q, entao f(m) > 0_k e f(1/n) > 0_k.
 
Agora, dado um racional positivo m/n (m,n inteiros), podemos assumir s.p.d.g. que m > 0 e n > 0 e, portanto, f(m/n) = f(m*1/n) = f(m)*f(1/n) > 0_k.
 
Ou seja, se a > 0 em Q, entao f(a) > 0_k em K.
 
[]s,
Claudio.
 
 
 
 
 
 
 
 


Yahoo! Messenger - Fale com seus amigos online. Instale agora!
TRANSIRE SVVM PECTVS MVNDOQVE POTIRI CONGREGATI EX TOTO ORBE MATHEMATICI OB SCRIPTA INSIGNIA TRIBVERE Fields Medal(John Charles Fields)
 
N.F.C. (Ne Fronti Crede)Yahoo! Mail - Participe da pesquisa global sobre o Yahoo! Mail. Clique aqui!

RE: [obm-l] Problema

2004-07-08 Thread Qwert Smith
Imagine um  quadrado ABCD  de lado a. Imagine agora dois móveis partindo do
mesmo ponto A. O móvel 1 faz os caminhos AB, BC, CD , DA , AB. O móvel 
2
percorre os caminhos AC, CA, AC, ou seja, só se move pela diagonal.  
Com
base nisso responda:
a) Se esses móveis tiverem mesma velocidade e sairem ao mesmo tempo do
vértice A , eles se encontrarão?. Justifique sua resposta.
b) Dê as possibilidades de encontro , sabendo que a diferença entre os 
pontos
de partida é inferior a 1 minuto.
c) Qual a diferença entre os tempos de partida do primeiro e do segundo
móvel, sendo que o encontro entre eles dá-se em 1 ano. considere que a 
distância AB
é percorrida em 1 minuto.
a)
Acho ki ja foi respondida
v = velocidade em 'a's por minuto, t = tempo em minutos,
d = distancia em 'a's
Se v1=v2 e t1=t2 entao d1=d2
Os possiveis pontos de encontro sao A e C
para A:
d1 = 4m (m inteiro >= 0)
d2 = 2n(2^1/2) (n inteiro >= 0)
para C:
d1 = 4m + 2
d2 = (2n + 1)(2^1/2)
Acho que fica bem obvio que d1 nunca sera igual a d2
b)
Nao entendi direito oke vc quer
d1 nunca sera igual a d2, mas se |d1 - d2| < 1, da pra
forcar um encontro:
Se d1 > d2, basta que 1 saia (d1 - d2) minutes
antes de 2.
Se d2 > d1, basta que 2 saia (d2 - d1) minutes
antes de 1.
c)
Aqui e preciso defenir ano em minutos... usando a definicao de que
1 ano = 365 dias = 365 * 24 horas = 365 * 24 * 60 minutos
d1 = 525600 (como e multiplo de 4 o encontro eh em A)
|d1 - d2| < 1 => |52600 - 2n(2^1/2)| < 1
n = 185828 e d2 ~= 525600.956
Logo basta que 2 saia ~ 0.956 minutos antes de 1
_
Get tips for maintaining your PC, notebook accessories and reviews in 
Technology 101. http://special.msn.com/tech/technology101.armx

=
Instruções para entrar na lista, sair da lista e usar a lista em
http://www.mat.puc-rio.br/~nicolau/olimp/obm-l.html
=


Re: [obm-l] Problema

2005-02-15 Thread Fábio Dias Moreira
benedito escreveu:
Quinze moedas de mesmo diâmetro são dispostas formando um triângulo 
eqüilátero. As faces de cada uma das moedas são pintadas ou de branco ou 
de preto. Prove que, qualquer que seja a pintura, existem três moedas de 
mesma cor cujos centros são vértices de um triângulo eqüilátero.
[...]
Suponha que não há triângulo equilátero e considere o tabuleiro:
.
   . .
  . a .
 . a a .
. . . . .
Os três quadrados marcados com a não podem ser da mesma cor. Suponha
s.p.d.g. que eles são pintados da seguinte forma:
.
   . .
  . O .
 . X X .
. . 1 . .
(Eu estou seguindo a convenção do Go -- O é branco, X é preto)
O ponto 1 deve ser branco:
.
   . .
  . O .
 2 X X 2
. . O . .
Os pontos 2 têm que ser pretos:
.
   . .
  . O .
 X X X X
. 3 O 3 .
Os pontos 3 têm que ser brancos:
.
   . .
  . * .
 X X X X
. * O * .
Mas então acabamos de formar um triângulo equilátero nos três pontos
marcados.
[]s,
--
Fábio Dias Moreira
=
Instruções para entrar na lista, sair da lista e usar a lista em
http://www.mat.puc-rio.br/~nicolau/olimp/obm-l.html
=


Re: [obm-l] Problema

2005-02-16 Thread Bruno Bruno
Olha, esse enunciado não é verdade. Podemos dipor as moedas da seguinte maneira:
 0
   0  0
 0  0
   0  0
 0  0
   0  0  0   0  0  0

Essa disposição é perfeitamente viável e não contraria o enuunciado
(15 moedas de mesmo diametro formando um triangulo equilatero).

Ora, usando essa convençao de X e O, podemos botar as moedas assim:

 O
   X   X
OX
  X O
   O  X
 X   X   O   X   O  O

Não formando assim, qualquer triangulo equilatero (não estou querendo
ser pentelho, mas é que realmente eu pensei nessa configuraçao antes
de pensar naquela onde estao todas amontoadas).

On Tue, 15 Feb 2005 21:53:09 -0300, Fábio Dias Moreira
<[EMAIL PROTECTED]> wrote:
> benedito escreveu:
> > Quinze moedas de mesmo diâmetro são dispostas formando um triângulo
> > eqüilátero. As faces de cada uma das moedas são pintadas ou de branco ou
> > de preto. Prove que, qualquer que seja a pintura, existem três moedas de
> > mesma cor cujos centros são vértices de um triângulo eqüilátero.
> > [...]
> 
> Suponha que não há triângulo equilátero e considere o tabuleiro:
> 
> .
>. .
>   . a .
>  . a a .
> . . . . .
> 
> Os três quadrados marcados com a não podem ser da mesma cor. Suponha
> s.p.d.g. que eles são pintados da seguinte forma:
> 
> .
>. .
>   . O .
>  . X X .
> . . 1 . .
> 
> (Eu estou seguindo a convenção do Go -- O é branco, X é preto)
> 
> O ponto 1 deve ser branco:
> 
> .
>. .
>   . O .
>  2 X X 2
> . . O . .
> 
> Os pontos 2 têm que ser pretos:
> 
> .
>. .
>   . O .
>  X X X X
> . 3 O 3 .
> 
> Os pontos 3 têm que ser brancos:
> 
> .
>. .
>   . * .
>  X X X X
> . * O * .
> 
> Mas então acabamos de formar um triângulo equilátero nos três pontos
> marcados.
> 
> []s,
> 
> --
> Fábio Dias Moreira
> 
> =
> Instruções para entrar na lista, sair da lista e usar a lista em
> http://www.mat.puc-rio.br/~nicolau/olimp/obm-l.html
> =
>

=
Instruções para entrar na lista, sair da lista e usar a lista em
http://www.mat.puc-rio.br/~nicolau/olimp/obm-l.html
=


Re: [obm-l] Problema

2005-02-16 Thread Fábio Dias Moreira
Bruno Bruno escreveu:
Olha, esse enunciado não é verdade. Podemos dipor as moedas da seguinte maneira:
 0
0 0
   0   0
  0 0
 0   0
0 0 0 0 0 0
Essa disposição é perfeitamente viável e não contraria o enuunciado
(15 moedas de mesmo diametro formando um triangulo equilatero).
Ora, usando essa convençao de X e O, podemos botar as moedas assim:
 O
X X
   O   X
  X O
 O   X
X X O X O O
Não formando assim, qualquer triangulo equilatero (não estou querendo
ser pentelho, mas é que realmente eu pensei nessa configuraçao antes
de pensar naquela onde estao todas amontoadas).
[...]
Bom, eu também pensei nessa distribuição inicialmente, e até achei
um contraexemplo -- por isso que eu fiz o problema com a
distribuição que eu mostrei. Mesmo assim, o seu exemplo contém
vários triângulos equiláteros monocromáticos (onde?).
[]s,
--
Fábio Dias Moreira
=
Instruções para entrar na lista, sair da lista e usar a lista em
http://www.mat.puc-rio.br/~nicolau/olimp/obm-l.html
=


Re: [obm-l] PROBLEMA!

2005-04-01 Thread Bernardo Freitas Paulo da Costa
Note que os maiores somam 15 e os menores, zero. Assim, você já tem
alguma coisa. Agora, veja quem pode somar 2 e quem pode somar 13... E
depois acho que vale o bom "chute".

Abraços,
-- 
Bernardo Freitas Paulo da Costa


On Apr 1, 2005 12:39 PM, Rafael Alfinito Ferreira <[EMAIL PROTECTED]> wrote:
> EU TENTEI, TENTEI E ATÉ AGORA NÃO ENTENDI
> 
> AÍ VAI:
> 
> DADOS 5 NÚMEROS, AS SOMA 2 A 2 SÃO: 0, 2, 4, 4, 6, 8, 9, 11, 13 E 15
> RESPECTIVAMENTE.
> DETERMINE OS NÚMEROS.
> 
> DESDE JÁ AGRADEÇO.
> 
> _
> MSN Messenger: converse online com seus amigos .
> http://messenger.msn.com.br
> 
> =
> Instruções para entrar na lista, sair da lista e usar a lista em
> http://www.mat.puc-rio.br/~nicolau/olimp/obm-l.html
> =
>

=
Instruções para entrar na lista, sair da lista e usar a lista em
http://www.mat.puc-rio.br/~nicolau/olimp/obm-l.html
=


Re: [obm-l] PROBLEMA!

2005-04-02 Thread Eduardo Wilner
  
  Oi Rafael.

  O problema, tal como formulado, não tem solução.  
  Senão vejamos: denominando os números, na ordem  
  crescente, a1,a2,a3,a4,a5,temos
  a1+a2=0  ou a1=-a2 <0 ; 
  a4+a5=15(*)  ; a2+a4=a1+a5=4 ;
  Assim, 0 -2
wrote:
> EU TENTEI, TENTEI E ATÉ AGORA NÃO ENTENDI
> 
> AÍ VAI:
> 
> DADOS 5 NÚMEROS, AS SOMA 2 A 2 SÃO: 0, 2, 4, 4, 6,
> 8, 9, 11, 13 E 15 
> RESPECTIVAMENTE.
> DETERMINE OS NÚMEROS.
> 
> DESDE JÁ AGRADEÇO.
> 
>
_
> MSN Messenger: converse online com seus amigos .  
> http://messenger.msn.com.br
> 
>
=
> Instruções para entrar na lista, sair da lista e
> usar a lista em
> http://www.mat.puc-rio.br/~nicolau/olimp/obm-l.html
>
=
> 





Yahoo! Acesso Grátis - Internet rápida e grátis. 
Instale o discador agora! http://br.acesso.yahoo.com/
=
Instruções para entrar na lista, sair da lista e usar a lista em
http://www.mat.puc-rio.br/~nicolau/olimp/obm-l.html
=


Re: [obm-l] PROBLEMA!

2005-04-04 Thread Carlos Gustavo Tamm de Araujo Moreira
   Caros Wilner e Rafael,
   a1=-a,a2=a,a1+a3=2, logo a3=2+a. As proximas menores somas possiveis sao
a1+a4 e a2+a3, e ambas devem valer 4, logo -a+a4=4, donde a4=4+a e 2+2a=4, 
donde a=1. Assim, a1=-1, a2=1, a3=3, a4=5 e, como devemos ter a4+a5=15,
a5=10. Os numeros sao, portanto: -1,1,3,5,10. E da' certo!
   Abracos,
Gugu

>
>  
>  Oi Rafael.
>
>  O problema, tal como formulado, não tem solução.  
>  Senão vejamos: denominando os números, na ordem  
>  crescente, a1,a2,a3,a4,a5,temos
>  a1+a2=0  ou a1=-a2 <0 ; 
>  a4+a5=15(*)  ; a2+a4=a1+a5=4 ;
>  Assim, 0 -2  Mas as ultimas duas são imcompatíveis com(*)!
>
>  De onde vc. tirou o problema? Deve haver algum
>engano.
>  Por falar nisso, o qye vc. quer dizer com
>RESPECTIVAMENTE?
>   
>  Abraços
>  
>  Wilner
>
>
>
>--- Rafael Alfinito Ferreira <[EMAIL PROTECTED]>
>wrote:
>> EU TENTEI, TENTEI E ATÉ AGORA NÃO ENTENDI
>> 
>> AÍ VAI:
>> 
>> DADOS 5 NÚMEROS, AS SOMA 2 A 2 SÃO: 0, 2, 4, 4, 6,
>> 8, 9, 11, 13 E 15 
>> RESPECTIVAMENTE.
>> DETERMINE OS NÚMEROS.
>> 
>> DESDE JÁ AGRADEÇO.
>> 
>>
>_
>> MSN Messenger: converse online com seus amigos .  
>> http://messenger.msn.com.br
>> 
>>
>=
>> Instruções para entrar na lista, sair da lista e
>> usar a lista em
>> http://www.mat.puc-rio.br/~nicolau/olimp/obm-l.html
>>
>=
>> 
>
>
>   
>   
>   
>Yahoo! Acesso Grátis - Internet rápida e grátis. 
>Instale o discador agora! http://br.acesso.yahoo.com/
>=
>Instruções para entrar na lista, sair da lista e usar a lista em
>http://www.mat.puc-rio.br/~nicolau/olimp/obm-l.html
>=
>

=
Instruções para entrar na lista, sair da lista e usar a lista em
http://www.mat.puc-rio.br/~nicolau/olimp/obm-l.html
=


Re: [obm-l] PROBLEMA!

2005-04-05 Thread Eduardo Wilner
 

Salve Gustavo.

Eu, de bobeira, omití o a3 nas somas que podem 
 dar 4.

Obrigado

Wilner


--- Carlos Gustavo Tamm de Araujo Moreira
<[EMAIL PROTECTED]> wrote:
>Caros Wilner e Rafael,
>a1=-a,a2=a,a1+a3=2, logo a3=2+a. As proximas
> menores somas possiveis sao
> a1+a4 e a2+a3, e ambas devem valer 4, logo -a+a4=4,
> donde a4=4+a e 2+2a=4, 
> donde a=1. Assim, a1=-1, a2=1, a3=3, a4=5 e, como
> devemos ter a4+a5=15,
> a5=10. Os numeros sao, portanto: -1,1,3,5,10. E da'
> certo!
>Abracos,
> Gugu
> 
> >
> >  
> >  Oi Rafael.
> >
> >  O problema, tal como formulado, não tem solução. 
> 
> >  Senão vejamos: denominando os números, na ordem  
> >  crescente, a1,a2,a3,a4,a5,temos
> >  a1+a2=0  ou a1=-a2 <0 ; 
> >  a4+a5=15(*)  ; a2+a4=a1+a5=4 ;
> >  Assim, 0 -2 >  Mas as ultimas duas são imcompatíveis com(*)!
> >
> >  De onde vc. tirou o problema? Deve haver algum
> >engano.
> >  Por falar nisso, o qye vc. quer dizer com
> >RESPECTIVAMENTE?
> >   
> >  Abraços
> >  
> >  Wilner
> >
> >
> >
> >--- Rafael Alfinito Ferreira <[EMAIL PROTECTED]>
> >wrote:
> >> EU TENTEI, TENTEI E ATÉ AGORA NÃO ENTENDI
> >> 
> >> AÍ VAI:
> >> 
> >> DADOS 5 NÚMEROS, AS SOMA 2 A 2 SÃO: 0, 2, 4, 4,
> 6,
> >> 8, 9, 11, 13 E 15 
> >> RESPECTIVAMENTE.
> >> DETERMINE OS NÚMEROS.
> >> 
> >> DESDE JÁ AGRADEÇO.
> >> 
> >>
>
>_
> >> MSN Messenger: converse online com seus amigos . 
> 
> >> http://messenger.msn.com.br
> >> 
> >>
>
>=
> >> Instruções para entrar na lista, sair da lista e
> >> usar a lista em
> >>
> http://www.mat.puc-rio.br/~nicolau/olimp/obm-l.html
> >>
>
>=
> >> 
> >
> >
> > 
> > 
> > 
> >Yahoo! Acesso Grátis - Internet rápida e grátis. 
> >Instale o discador agora!
> http://br.acesso.yahoo.com/
>
>=
> >Instruções para entrar na lista, sair da lista e
> usar a lista em
> >http://www.mat.puc-rio.br/~nicolau/olimp/obm-l.html
>
>=
> >
> 
>
=
> Instruções para entrar na lista, sair da lista e
> usar a lista em
> http://www.mat.puc-rio.br/~nicolau/olimp/obm-l.html
>
=
> 





Yahoo! Acesso Grátis - Internet rápida e grátis. 
Instale o discador agora! http://br.acesso.yahoo.com/
=
Instruções para entrar na lista, sair da lista e usar a lista em
http://www.mat.puc-rio.br/~nicolau/olimp/obm-l.html
=


Re: [obm-l] problema

2005-06-20 Thread Renato Lira
Seja H o ortocentro do triangulo.
Note que o #BOHM é inscritível numa circunferencia, logo  os angulos MOH=MBN=40
é facil notar que HCN=40
o #OCNH é inscritivel logo HCO=HON=40
Finalmente concluimos que MON=HOM+HON=40+40=80 
On 6/20/05, Guilherme Neves <[EMAIL PROTECTED]> wrote:


Dado um triangulo qualquer ABC, e M , N e O os pés das alturas sobre os lados AB, AC e BC, respectivamente. Se o angulo ABN vale 40º, quanto vale o angulo MON?

MSN Busca: fácil, rápido, direto ao ponto. Encontre o que você quiser. Clique aqui. = Instruções para entrar na lista, sair da lista e usar a lista em 
http://www.mat.puc-rio.br/~nicolau/olimp/obm-l.html = 



Re: [obm-l] Problema

2005-06-25 Thread Leo



Desenhe a figura com todos os dadosPor Pitágora 
vc saberá inúmeras coisas como:
AB=4 e BC=3 implica em BD=5... a pependicular 'a 
diagonal BD por A vale 2,4, de fato eh altura do triângulo 3,4,5 asim tb teremos 
DH=1,8   HM=1,6    MB=1,6   DN=2   
e   NC=2
Olhando para o triângulo DMN temos os valores para 
o lado DM =1,8+1,6=3,4   e DN=2   e MN=??  e o ângulo 
MDN= fi
Pela Lei dos cossenos temos o lado MN  
MN^2=3,4^2 +2^2 - 2*3,4*2*cos(fi)   mas 
cos(fi)=BC/DB=3/4=0,75
segue a resposta..MN=raiz_quad(5,36)~=2,31... 

abraço..
Leonardo Borges Avelino

  - Original Message - 
  From: 
  Lincoln 
  To: obm-l@mat.puc-rio.br 
  Sent: Friday, May 07, 2004 12:26 AM
  Subject: [obm-l] Problema
  
  Alguém pode me dar uma ajuda neste 
  problema?
   
  
  Seja 
  ABCD um retângulo de lados AB = 4 e BC =3.  A perpendicular à diagonal BD traçada por A corta BD no ponto H. Chamamos de M o ponto médio de BH e de N o ponto médio de CD. Calcule a medida do segmento MN. 
   
   


Re: [obm-l] Problema

2005-06-28 Thread Marcio M Rocha

Lincoln escreveu:


Alguém pode me dar uma ajuda neste problema?
 

Seja /ABCD/ um retângulo de lados /AB/ = 4 e /BC/ =3.  A perpendicular 
à diagonal /BD/ traçada por /A/ corta /BD/ no ponto /H/. Chamamos de 
/M/ o ponto médio de /BH/ e de /N/ o ponto médio de /CD/. Calcule a 
medida do segmento /MN/.


 

 


Lincoln,
Escrevi rápido e sem muita organização. Veja se você entende e se está 
tudo OK.


BD = 5 e AH = 2,4. A perpendicular ao segmento AH, passando por M, 
intersecta AB no ponto P. Desse modo, MP = 1,2 e P é o médio de AB. Os 
segmentos PN e BD cortam-se ao meio, num ponto Q.  Seja R um ponto de 
BD, simétrico de M em relação a Q. Os triângulos PMQ e NRQ são 
congruentes. Desse modo, MN^2 = RN^2 + RM^2, ou seja, MN^2 = (1,2)^2 + 
(1,8)^2. Daí, MN = 3*sqrt(13)/5.


[]s,

Márcio.
=
Instruções para entrar na lista, sair da lista e usar a lista em
http://www.mat.puc-rio.br/~nicolau/olimp/obm-l.html
=


Re: [obm-l] Problema

2009-09-22 Thread Lucas Prado Melo
2009/9/22 Luís Eduardo Háteras 

>  Sou novo nessa lista e estou com dúvida nesse exercício, alguém saberia
> resolver ? E alguém sabe como me explicar porque não consegui compreender
> como resolver.
>
Primeira coisa tempo = comprimento / velocidade.
É preciso igualar o tempo que o trem leva para ultrapassar cada um e o tempo
que cada leva para chegar nas posições.

Por exemplo, o trem percorre um comprimento de C-30 (onde C é seu próprio
comprimento) para ultrapassar Bruno e Bruno percorre 30m neste mesmo tempo,
então:
(C-30)/Vt = 30/Vp

Onde Vt é velocidade do trem e Vp é velocidade das pessoas.

Basta encontrar a outra equação e resolver.


RE: [obm-l] Problema

2009-09-24 Thread Albert Bouskela
Olá,

 

Este problema é muito (muito, mesmo!) fácil: - considere uma P.A. com os
seguintes parâmetros:

 

a(1) = 100 (1º termo) / a(900)=999 (último termo) / r=1

 

E calcule a soma dos termos desta P.A.

 

Um problema mais interessante consiste em calcular a soma de todos os
algarismos de todos os números naturais formados por 3 algarismos – tente
resolvê-lo.

 

A.

  bousk...@msn.com

 

From: owner-ob...@mat.puc-rio.br [mailto:owner-ob...@mat.puc-rio.br] On
Behalf Of Paulo Barclay Ribeiro
Sent: Thursday, September 24, 2009 1:55 PM
To: obm-l@mat.puc-rio.br
Subject: [obm-l] Problema

 


Prezados, 

 

Peço uma ajuda (orientação)na resolução do seguinte problema:

Qual o valor da soma de todos os numeros naturais de três algarismos?

 

Desde  já agradeço a gentileza

 

Paulo Barclay 

 

  _  

Veja quais são os assuntos do momento no Yahoo! + Buscados: Top
  10 - Celebridades
  - Música
  - Esportes
 



RE: [obm-l] Problema

2009-09-24 Thread Helton Duarte

Olá Paulo,

A soma dos números naturais de 3 algarismos é a seguinte:
100 + 101 + ... + 998 + 999 = (100+999)900/2 = 494550
Esse problema foi resolvido usando a fórmula de soma dos n primeiros termos de 
uma PA, caso não a conheça, segue abaixo a demonstração:

S = 100 + 101 + ... + 998 + 999
invertendo os termos
S = 999 + 998 + ... + 101 + 100
S + S = (100 + 999) + (101 + 998) + ... + (998 + 101) + (999 + 100)
2 S = (100+999)900
=> S = (100+999)900/2 <=


Atenciosamente,

Helton de Melo Duarte

heltonduarte.com

twitter.com/heltonduarte



Date: Thu, 24 Sep 2009 09:54:40 -0700
From: paulobarc...@yahoo.com.br
Subject: [obm-l] Problema
To: obm-l@mat.puc-rio.br

Prezados, 
 
Peço uma ajuda (orientação)na resolução do seguinte problema:
Qual o valor da soma de todos os numeros naturais de três algarismos?
 
Desde  já agradeço a gentileza
 
Paulo Barclay 



  Veja quais são os assuntos do momento no Yahoo! + Buscados: Top 10 - 
Celebridades - Música - Esportes   
_
Descubra todas as novidades do novo Internet Explorer 8
http://brasil.microsoft.com.br/IE8/mergulhe/?utm_source=MSN%3BHotmail&utm_medium=Tagline&utm_campaign=IE8

Re: [obm-l] Problema

2009-09-24 Thread Hugo Fernando Marques Fernandes
Soma dos termos de uma P.A.

S = n*(a1+aN) / 2

n = 999 - 100 + 1 = 900

a1 = 100

aN = 999

S = 900*(100+999)/2 = 494550

Abraços.
Hugo.

2009/9/24 Paulo Barclay Ribeiro 

> Prezados,
>
> Peço uma ajuda (orientação)na resolução do seguinte problema:
> Qual o valor da soma de todos os numeros naturais de três algarismos?
>
> Desde  já agradeço a gentileza
>
> Paulo Barclay
>
> --
> Veja quais são os assuntos do momento no Yahoo! + Buscados: Top 
> 10-
> Celebridades-
> Música-
> Esportes
>


RE: [obm-l] Problema

2009-09-28 Thread Paulo Barclay Ribeiro
Prezados, mais uma vez muito obrigado a todos por atenderem a minha dificuldade.
peço desculpas em responder.è que estou com problemas no Velox, e meu micro não 
é lá essas coisas.
 
Um abraço
 
Paulo Barclay.

--- Em qui, 24/9/09, Helton Duarte  escreveu:


De: Helton Duarte 
Assunto: RE: [obm-l] Problema
Para: "OBM - Lista" 
Data: Quinta-feira, 24 de Setembro de 2009, 19:23




Olá Paulo,

A soma dos números naturais de 3 algarismos é a seguinte:
100 + 101 + ... + 998 + 999 = (100+999)900/2 = 494550
Esse problema foi resolvido usando a fórmula de soma dos n primeiros termos de 
uma PA, caso não a conheça, segue abaixo a demonstração:

S = 100 + 101 + ... + 998 + 999
invertendo os termos
S = 999 + 998 + ... + 101 + 100
S + S = (100 + 999) + (101 + 998) + ... + (998 + 101) + (999 + 100)
2 S = (100+999)900
=> S = (100+999)900/2 <=


Atenciosamente,

Helton de Melo Duarte
heltonduarte.com
twitter.com/heltonduarte





Date: Thu, 24 Sep 2009 09:54:40 -0700
From: paulobarc...@yahoo.com.br
Subject: [obm-l] Problema
To: obm-l@mat.puc-rio.br






Prezados, 
 
Peço uma ajuda (orientação)na resolução do seguinte problema:
Qual o valor da soma de todos os numeros naturais de três algarismos?
 
Desde  já agradeço a gentileza
 
Paulo Barclay 


Veja quais são os assuntos do momento no Yahoo! + Buscados: Top 10 - 
Celebridades - Música - Esportes 


Conheça os novos produtos Windows Live. Clique aqui! 


  

Veja quais são os assuntos do momento no Yahoo! +Buscados
http://br.maisbuscados.yahoo.com

Re: [obm-l] Problema

2009-11-16 Thread Paulo Santa Rita
Ola benedito e demais
colegas desta lista ... OBM-L,
(escreverei sem acentos)


Seja An o conjunto de todos os triangulos cujos lados são numeros
inteiros menores ou iguais a N.  Entao, claramente,  An-1 esta contido
em An ...  Significa isso que - representando por (A) o numero de
elementos do conjunto A - podemos por  :

(An) = (An-1) + ( Bn)

onde Bn e o conjunto dos elementos de An que não estao em An-1. E
facil ver que os elementos de Bn são todos os triangulos de An nos
quais ao menos um lado vale N.

Quantos elementos tem Bn ?

Bom, a principio, e facil ver que em Bn esta o triangulo equilatero de
lado N. E igualmente facil perceber que Bn congrega tambem todos os
triangulos isosceles e nao-equilateros nos quais dois de seus lados
valem N, a saber, os triangulos {N,N,1}, {N,N, 2}, ... {N,N,N-1}.
Computando tudo isso temos N triangulos. Portanto :

(Bn)= N + (Cn)

onde Cn e o conjunto de todos os triangulos de An nos quais um, e
somente um, dos lados vale N, a saber, os escalenos cujo maior lado
vale N e os isosceles não equilateros cujos lados iguais são menores
que N. Os triangulos de lados {N,N-1,N-2} e {N,N-1,N-1} são exemplos
validos para esta duas classes.

Quantos são os elementos de Cn ?

Todos os elementos de Cn tem um único lado medindo N e, alem disso,
este lado e o maior lado. Isto implica que se representarmos
genericamente um destes triangulos por {N, A, B}, devera ocorrer :

1)A+B >=  N+1,  pois devemos ter N < A+B
2)A+B =< 2N-2,  pois A < N e B < N

significa isso que os elementos de Cn estao agrupados em classes
disjuntas, nas quais todos os elementos de uma mesma classe tem o
mesmo perimetro. Enumerando os elementos da classe {N,A,B} na qual
A+B=N+1 ate a enumeracao da classe {N,A,B} na qual A+B=2N-2 teremos
totalizado todos os elementos de Cn.

Seja portanto D2p ( indice “2p” ) a classe de triangulos {N, A, B} de
Cn na qual A+B=2p. Temos que 2p=N+1, N+2,..., 2N-2. Fixando uma D2p
qualquer, podemos IMAGINAR que cada elemento desta D2p e uma sequencia
de tres numeros, ordenados da esquerda para a direita, do maior lado
para o menor lado.

Agora, IMAGINE que as sequencias ordenadas descritas acima estao elas
mesmas ordenadas de forma decrescente pelo elemento  central ( o
segundo termo de cada 3-sequencia ). O que vemos ?

(N,  (N-1)-0,  (2p-N+1)+0)
(N,  (N-1)-1,  (2p-N+2)+1)
(N,  (N-3)-2,  (2p-N+3)+2)
...

E ate onde podemos descer ? Ate X tal que (N-1)-X  >= 2p-(N-1)+X pois
se (N-1)-X < p-(N-1)+X claramente que o triangulo {N,(N-1-X,p-(N-1)+X}
sera igual a algum dos anteriores, já computado. Assim :

X =< (N-1) – p.

Para considerar o valor X=0, fazemos: 1 + X  =<  N-p. E como 1+X deve
ser inteiro, para não dependermos da paridade de N, colocamos :

(D2p) = 1+X = piso(N – p)

De tudo que vimos chegamos a :
(An) = (An-1)  +  N  +  (Dn+1)  +  (Dn+2)  +  ...  +  (D2n-3)  +  (D2n-2)
o que resolve o problema original formulado pelo Benedito.

Agora, facamos alguns calculos praticos.

N=1  => A1= 1
Obvio, pois apenas o triangulo {1,1,1} atende as condicoes de simetria
do problema.

N=2  => A2= 3
Obvio, pois alem do triangulo {1,1,1} somente os triangulos {2,2,1} e
{2,2,2} interessam.

N=3 => A3 = A2 + 3 + D4 = 3 + 3 + piso(3-2) = 3 + 3 + 1 = 7
Os 4 novos triangulos são {3,3,3}, {3,3,2},  {3,3,1} e {3,2,2}

N=4 => A4=A3 +4+D5+D6 = 7 + 4 + piso(4 - 2,5) + piso(4 - 3)=13
Os 6 novos triangulos são {4,4,4},{4,4,3},{4,4,2},{4,4,1},{4,3,2} e {4,3,3}

Agora, vamos considerar com mais atencao a expressao que fornece o
numero de elementos de D2p:

(D2p)=piso(N - p)

Esta expressao e bonita ? Não sei … O que voces acham ? Eu tenho
minhas duvidas … A funcao “piso” da uma certa assimetria a formula,
tornando-a carrancuda. Ela e decididamente uma mulher com veu, mas eu
vou apostar e continuar esta viagem com ela para ver aonde ela me
conduz … Se ela for bela, ela sera fertil !

Esta formula nos diz quantos triangulos de lados inteiros positivos
tem perimetro p, com as limitacoes :

1)Um unico lado deve valer N
2)N+1 =<2p   =< 2N-2

E se quisessemos encontrar “todos os triangulos de lados inteiros que
tenham perimetro 2p”, independente das limitacoes acima ?  Nos temos
elementos suficientes para resolver esta questao diretamente ?

Temos. Eis como :

Se um triangulo de lados inteiros tem perimetro 2p, o maior lado
possivel deve ser L= p -1 se 2p e par; deve ser L=p – 0,5 se 2p e
impar, pois em qualquer triangulo, o maior lado deve ser menor que a
soma dos outros dois. Sintetizamos tudo isso pondo L = piso(p – 0,5).

E o menor maior lado possivel ? E claro que se M e o menor maior lado
possivel deve ocorrer que 3M  >= 2p. Assim, o menor maior lado
possivel e o menor M tal que 3M >= 2p  =>  M=teto(2p/3).

Usando a notacao  Si{ A,B : f(i) }=f(A) + f(A+1) + … + f(B-1) + f(B)
para representar o somatorio e aplicando a expressao D2p=piso(N-p) aos
resultados acima, chegamos a :

T(2p) = ( L – M + 1) + Si{ M , L : piso( (3i/2) – p ) }
onde M=teto(2p/3), L=piso(p-0,5) e T(2p) e o numero de triangulos com
lad

Re: [obm-l] Problema

2009-11-17 Thread Benedito

Obrigado Paulo.
Valeu
Benedito

- Original Message - 
From: "Paulo Santa Rita" 

To: 
Sent: Friday, November 13, 2009 12:49 PM
Subject: Re: [obm-l] Problema



Ola benedito e demais
colegas desta lista ... OBM-L,
(escreverei sem acentos)


Seja An o conjunto de todos os triangulos cujos lados são numeros
inteiros menores ou iguais a N.  Entao, claramente,  An-1 esta contido
em An ...  Significa isso que - representando por (A) o numero de
elementos do conjunto A - podemos por  :

(An) = (An-1) + ( Bn)

onde Bn e o conjunto dos elementos de An que não estao em An-1. E
facil ver que os elementos de Bn são todos os triangulos de An nos
quais ao menos um lado vale N.

Quantos elementos tem Bn ?

Bom, a principio, e facil ver que em Bn esta o triangulo equilatero de
lado N. E igualmente facil perceber que Bn congrega tambem todos os
triangulos isosceles e nao-equilateros nos quais dois de seus lados
valem N, a saber, os triangulos {N,N,1}, {N,N, 2}, ... {N,N,N-1}.
Computando tudo isso temos N triangulos. Portanto :

(Bn)= N + (Cn)

onde Cn e o conjunto de todos os triangulos de An nos quais um, e
somente um, dos lados vale N, a saber, os escalenos cujo maior lado
vale N e os isosceles não equilateros cujos lados iguais são menores
que N. Os triangulos de lados {N,N-1,N-2} e {N,N-1,N-1} são exemplos
validos para esta duas classes.

Quantos são os elementos de Cn ?

Todos os elementos de Cn tem um único lado medindo N e, alem disso,
este lado e o maior lado. Isto implica que se representarmos
genericamente um destes triangulos por {N, A, B}, devera ocorrer :

1)A+B >=  N+1,  pois devemos ter N < A+B
2)A+B =< 2N-2,  pois A < N e B < N

significa isso que os elementos de Cn estao agrupados em classes
disjuntas, nas quais todos os elementos de uma mesma classe tem o
mesmo perimetro. Enumerando os elementos da classe {N,A,B} na qual
A+B=N+1 ate a enumeracao da classe {N,A,B} na qual A+B=2N-2 teremos
totalizado todos os elementos de Cn.

Seja portanto D2p ( indice “2p” ) a classe de triangulos {N, A, B} de
Cn na qual A+B=2p. Temos que 2p=N+1, N+2,..., 2N-2. Fixando uma D2p
qualquer, podemos IMAGINAR que cada elemento desta D2p e uma sequencia
de tres numeros, ordenados da esquerda para a direita, do maior lado
para o menor lado.

Agora, IMAGINE que as sequencias ordenadas descritas acima estao elas
mesmas ordenadas de forma decrescente pelo elemento  central ( o
segundo termo de cada 3-sequencia ). O que vemos ?

(N,  (N-1)-0,  (2p-N+1)+0)
(N,  (N-1)-1,  (2p-N+2)+1)
(N,  (N-3)-2,  (2p-N+3)+2)
...

E ate onde podemos descer ? Ate X tal que (N-1)-X  >= 2p-(N-1)+X pois
se (N-1)-X < p-(N-1)+X claramente que o triangulo {N,(N-1-X,p-(N-1)+X}
sera igual a algum dos anteriores, já computado. Assim :

X =< (N-1) – p.

Para considerar o valor X=0, fazemos: 1 + X  =<  N-p. E como 1+X deve
ser inteiro, para não dependermos da paridade de N, colocamos :

(D2p) = 1+X = piso(N – p)

De tudo que vimos chegamos a :
(An) = (An-1)  +  N  +  (Dn+1)  +  (Dn+2)  +  ...  +  (D2n-3)  +  (D2n-2)
o que resolve o problema original formulado pelo Benedito.

Agora, facamos alguns calculos praticos.

N=1  => A1= 1
Obvio, pois apenas o triangulo {1,1,1} atende as condicoes de simetria
do problema.

N=2  => A2= 3
Obvio, pois alem do triangulo {1,1,1} somente os triangulos {2,2,1} e
{2,2,2} interessam.

N=3 => A3 = A2 + 3 + D4 = 3 + 3 + piso(3-2) = 3 + 3 + 1 = 7
Os 4 novos triangulos são {3,3,3}, {3,3,2},  {3,3,1} e {3,2,2}

N=4 => A4=A3 +4+D5+D6 = 7 + 4 + piso(4 - 2,5) + piso(4 - 3)=13
Os 6 novos triangulos são {4,4,4},{4,4,3},{4,4,2},{4,4,1},{4,3,2} e 
{4,3,3}


Agora, vamos considerar com mais atencao a expressao que fornece o
numero de elementos de D2p:

(D2p)=piso(N - p)

Esta expressao e bonita ? Não sei … O que voces acham ? Eu tenho
minhas duvidas … A funcao “piso” da uma certa assimetria a formula,
tornando-a carrancuda. Ela e decididamente uma mulher com veu, mas eu
vou apostar e continuar esta viagem com ela para ver aonde ela me
conduz … Se ela for bela, ela sera fertil !

Esta formula nos diz quantos triangulos de lados inteiros positivos
tem perimetro p, com as limitacoes :

1)Um unico lado deve valer N
2)N+1 =<2p   =< 2N-2

E se quisessemos encontrar “todos os triangulos de lados inteiros que
tenham perimetro 2p”, independente das limitacoes acima ?  Nos temos
elementos suficientes para resolver esta questao diretamente ?

Temos. Eis como :

Se um triangulo de lados inteiros tem perimetro 2p, o maior lado
possivel deve ser L= p -1 se 2p e par; deve ser L=p – 0,5 se 2p e
impar, pois em qualquer triangulo, o maior lado deve ser menor que a
soma dos outros dois. Sintetizamos tudo isso pondo L = piso(p – 0,5).

E o menor maior lado possivel ? E claro que se M e o menor maior lado
possivel deve ocorrer que 3M  >= 2p. Assim, o menor maior lado
possivel e o menor M tal que 3M >= 2p  =>  M=teto(2p/3).

Usando a notacao  Si{ A,B : f(i) }=f(A) + f(A+1) + … + f(B-1)

Re: [obm-l] Problema

2012-02-06 Thread Eduardo Wilner
A velocidade da nave que viaja pela diagonal eh o triplo da que viaja pela 
aresta, percorrendo uma diastancia \sqrt3 vezes a percorrida pela segunda, 
portanto num intervalo de tempo menor.
Como elas terminam as "viagens" no mesmo instante t=0, no instante t=-1 ( no 
exemplo da resolução ) , quando a nave "alfa" estah partindo de B, a nave 
"beta" ainda não está partindo ou estaria virtualmente num ponto aquem de A, no 
caso (1-\sqrt3)(1,1,1).

Me parece que o problema seria mais "Olímpico" (e talvez fosse a intenção do 
autor, que se distraiu) se a nave que viaja pela diagonal tivesse velocidade 
\srt3 vezes a que viaja pela aresta, (quando sua pergunta teria "a ver", como 
dizem os jovens) e a distancia seria minima no meio do percurso, igual a 
(sqrt2)/2... 

[ ]'s
 

--- Em dom, 5/2/12, Henrique Rennó  escreveu:

De: Henrique Rennó 
Assunto: [obm-l] Problema
Para: "obm-l" 
Data: Domingo, 5 de Fevereiro de 2012, 12:41

Oi, boa tarde.

A solução do problema 1 da primeira fase do nível universitário na eureka 34 
página 60 
(http://www.obm.org.br/export/sites/default/revista_eureka/docs/eureka34.pdf) 
apresenta a função de posição considerando tempos inicial -1 e final 0, sendo a 
função do objeto mais rápido dada por B(t) = (1,1,1) + rq(3)*t*(1,1,1), onde rq 
é a raíz quadrada. Considerando t = -1 na equação, temos B(-1) = (1-rq(3), 
1-rq(3), 1-rq(3)), que é diferente da posição inicial (0,0,0). Outra dúvida é 
como ficariam as funções se considerarmos como tempo inicial e final os valores 
0 e 1, respectivamente.


Obrigado

-- 
Henrique




Re: [obm-l] Problema

2012-02-07 Thread Henrique Rennó
Eu assumi erroneamente que as naves partiam dos pontos iniciais juntas e
chegavam em seus pontos finais juntas no mesmo intervalo de tempo. Essa
distância rq(3) seria percorrida em cada uma das direções pela nave mais
rápida.

2012/2/6 Eduardo Wilner 

> A velocidade da nave que viaja pela diagonal eh o triplo da que viaja pela
> aresta, percorrendo uma diastancia \sqrt3 vezes a percorrida pela segunda,
> portanto num intervalo de tempo menor.
> Como elas terminam as "viagens" no mesmo instante t=0, no instante t=-1 (
> no exemplo da resolução ) , quando a nave "alfa" estah partindo de B, a
> nave "beta" ainda não está partindo ou estaria virtualmente num ponto
> aquem de A, no caso (1-\sqrt3)(1,1,1).
>
> Me parece que o problema seria mais "Olímpico" (e talvez fosse a intenção
> do autor, que se distraiu) se a nave que viaja pela diagonal tivesse
> velocidade \srt3 vezes a que viaja pela aresta, (quando sua pergunta teria
> "a ver", como dizem os jovens) e a distancia seria minima no meio do
> percurso, igual a (sqrt2)/2...
>
> [ ]'s
>
>
> --- Em *dom, 5/2/12, Henrique Rennó * escreveu:
>
>
> De: Henrique Rennó 
> Assunto: [obm-l] Problema
> Para: "obm-l" 
> Data: Domingo, 5 de Fevereiro de 2012, 12:41
>
>
> Oi, boa tarde.
>
> A solução do problema 1 da primeira fase do nível universitário na eureka
> 34 página 60 (
> http://www.obm.org.br/export/sites/default/revista_eureka/docs/eureka34.pdf)
> apresenta a função de posição considerando tempos inicial -1 e final 0,
> sendo a função do objeto mais rápido dada por B(t) = (1,1,1) +
> rq(3)*t*(1,1,1), onde rq é a raíz quadrada. Considerando t = -1 na equação,
> temos B(-1) = (1-rq(3), 1-rq(3), 1-rq(3)), que é diferente da posição
> inicial (0,0,0). Outra dúvida é como ficariam as funções se considerarmos
> como tempo inicial e final os valores 0 e 1, respectivamente.
>
> Obrigado
>
> --
> Henrique
>
>


-- 
Henrique


Re: [obm-l] Problema

2012-02-07 Thread Bernardo Freitas Paulo da Costa
2012/2/7 Bob Roy :
> 0lá ,
1 cá.

> Poderiam me ajudar na questão  a seguir  ?
>
> Em uma circunferência colocamos os números 5, 2,3,0, 5 e 6 (por exemplo
> nesta ordem  no sentido horário) .A cada momento escolho  um número qualquer
> e adiciono uma unidade a ele e aos dois vizinhos .É posível em algum momento
> todos ficarem iguais ? se possível , como devemos proceder ?
Bom, uma idéia pra começar esse tipo de problemas é olhar paridade.
Assim, temos I, P, I, P, I, P no círculo. Começando pelos 3 primeiros,
chegamos em P, I, P, P, I, P. Em seguida, fazemos no 2°, 3° e 4° dando
P, P, I, I, I, P. Com mais uma (a partir do 3°, por exemplo), todos
têm a mesma paridade. Bom, isso quer dizer que dá pra acertar as
paridades... Vamos ver o que a gente conseguiu :

5, 2, 3, 0, 5, 6
6, 3, 4, 0, 5, 6
6, 4, 5, 1, 5, 6
6, 4, 6, 2, 6, 6

Hum, dá pra chutar que "não vai dar" porque o 4, 6, 2 no meio é "muito
pequeno" comparado ao 6, 6, 6 das bordas. Mas teria que provar...

Uma variante seria tentar módulo 3. Veja que "somar 1 em três números"
não altera a soma módulo 3 dos seus elementos. O "problema" é que 5 +
2 + 3 + 0 + 5 + 6 = 21, que é múltiplo de 3.

Uma última idéia: note que você pode fazer "translações", ou seja, 6,
4, 6, 2, 6, 6 é a mesma coisa que 4, 2, 4, 0, 4, 4. Repare também que
as somas "3 a 3" são SEMPRE complementares. Deixa eu dizer isso
direito: considere as somas a partir do 1°, 2°, ..., 6° elemento. No
caso que a gente tem, isso dá:
10, 6, 8, 8, 12, 10,
e as somas 10 + 8 = 6 + 12 = 8 + 10 (porque afinal de contas a soma
dos 6 é sempre a mesma !!)

Espero que ajude !
-- 
Bernardo Freitas Paulo da Costa

=
Instruções para entrar na lista, sair da lista e usar a lista em
http://www.mat.puc-rio.br/~obmlistas/obm-l.html
=


RE: [obm-l] Problema

2012-02-07 Thread João Maldonado

Sendo  "a" a quantidade de vezes que foi realizada a operação no número 5, "b"  
no 2, "c" no 3 ...
Montamos  o sistema
5+f+a+b = 2 + a + b + c = 3 + b + c + d = 0 + c + d + e = 5 + d + e + f = 6 + e 
+ f + a

que é  equivalente a:
a-d = 1b-e = -3c-f = 5d-a = 1e-b = -1f-c = -3

Podemos  facilmente resolvê-lo  por Rouché-Capelli
a matriz incompleta será: 
 1  0  0 -1  0  0 0  1  0  0 -1  0 0  0  1  0  0 -1-1  0  0  1  0  0 0 -1  0  0 
 1  0 0  0 -1  0  0  1
Somando a primeira linha com a quarta obtemos uma linha nula -> determinante 
igual a zeroEliminando a última linha e coluna:
 1  0  0 -1  0 0  1  0  0 -1 0  0  1  0  0-1  0  0  1  0 0 -1  0  0  1
Somando a primeira linha com a quarta obtemos uma linha nula -> determinante 
igual a zeroEliminando a última linha e coluna: 1  0  0 -1 0  1  0  0 0  0  1  
0-1  0  0  1
Somando a primeira linha com a quarta obtemos uma linha nula -> determinante 
igual a zeroEliminando a última linha e coluna:
 1  0  0 0  1  0 0  0  1
Temos determinante 1,  logo o determinante principal da matriz é 1
A  matriz tem 3 determinantes característicos
Analisando o primeiro
 1  0  0  1 0  1  0 -3 0  0  1  5-1  0  0  1
Somando a primeira coluna com a última:
 1  0  0  2 0  1  0 -3 0  0  1  5-1  0  0  0
por  Laplace  D = (-1)(-1).
 0  0  2 1  0 -3 0  1  5
por Laplace D = (-1)(-1)(-1)(1).
 0  2 1  5
D = 2 != 0
Logo não existe solução para o sistema
[]'sJoão
Date: Tue, 7 Feb 2012 17:09:26 -0300
Subject: [obm-l] Problema
From: bob...@globo.com
To: obm-l@mat.puc-rio.br

0lá ,

Poderiam me ajudar na questão  a seguir  ?

Em uma circunferência colocamos os números 5, 2,3,0, 5 e 6 (por exemplo nesta 
ordem  no sentido horário) .A cada momento escolho  um número qualquer e 
adiciono uma unidade a ele e aos dois vizinhos .É posível em algum momento 
todos ficarem iguais ? se possível , como devemos proceder ?


Agradeço desde já qualquer  resposta 

Bob 


  

Re: [obm-l] Problema

2012-02-07 Thread Bernardo Freitas Paulo da Costa
2012/2/8 João Maldonado :
> Sendo  "a" a quantidade de vezes que foi realizada a operação no número 5,
> "b"  no 2, "c" no 3 ...
>
> Montamos  o sistema
>
> 5+f+a+b = 2 + a + b + c = 3 + b + c + d = 0 + c + d + e = 5 + d + e + f = 6
> + e + f + a
>
> que é  equivalente a:
>
> a-d = 1
> b-e = -3
> c-f = 5
> d-a = 1
> e-b = -1
> f-c = -3

Parô parô parô !!
a - d = 1
d - a = 1 (absurdo)

b - e = -3
e - b = -1 (outro absurdo)

c - f = 5
f - c = -3 (aiaiaiaiai...)

Genial, é assim que dá pra interpretar "a diferença dos 666 e o 462"
que eu botei no mail anterior. Valeu João por montar o sistema e não
ser preguiçoso como eu!

-- 
Bernardo Freitas Paulo da Costa

=
Instruções para entrar na lista, sair da lista e usar a lista em
http://www.mat.puc-rio.br/~obmlistas/obm-l.html
=


RE: [obm-l] Problema

2012-02-07 Thread Vitor Alves


João Maldonado,não li sua solução por completo,mas vê-se facilmente que vc 
errou no sistema ou vc naum prestou atenção q naum seria necessário vc fazer o 
sistema,pois de a-d=1 e d-a=1 tem-se um absurdo.AbraçosVitor de Lima Alves
From: joao_maldona...@hotmail.com
To: obm-l@mat.puc-rio.br
Subject: RE: [obm-l] Problema
Date: Tue, 7 Feb 2012 21:15:36 -0200







Sendo  "a" a quantidade de vezes que foi realizada a operação no número 5, "b"  
no 2, "c" no 3 ...
Montamos  o sistema
5+f+a+b = 2 + a + b + c = 3 + b + c + d = 0 + c + d + e = 5 + d + e + f = 6 + e 
+ f + a

que é  equivalente a:
a-d = 1b-e = -3c-f = 5d-a = 1e-b = -1f-c = -3

Podemos  facilmente resolvê-lo  por Rouché-Capelli
a matriz incompleta será: 
 1  0  0 -1  0  0 0  1  0  0 -1  0 0  0  1  0  0 -1-1  0  0  1  0  0 0 -1  0  0 
 1  0 0  0 -1  0  0  1
Somando a primeira linha com a quarta obtemos uma linha nula -> determinante 
igual a zeroEliminando a última linha e coluna:
 1  0  0 -1  0 0  1  0  0 -1 0  0  1  0  0-1  0  0  1  0 0 -1  0  0  1
Somando a primeira linha com a quarta obtemos uma linha nula -> determinante 
igual a zeroEliminando a última linha e coluna: 1  0  0 -1 0  1  0  0 0  0  1  
0-1  0  0  1
Somando a primeira linha com a quarta obtemos uma linha nula -> determinante 
igual a zeroEliminando a última linha e coluna:
 1  0  0 0  1  0 0  0  1
Temos determinante 1,  logo o determinante principal da matriz é 1
A  matriz tem 3 determinantes característicos
Analisando o primeiro
 1  0  0  1 0  1  0 -3 0  0  1  5-1  0  0  1
Somando a primeira coluna com a última:
 1  0  0  2 0  1  0 -3 0  0  1  5-1  0  0  0
por  Laplace  D = (-1)(-1).
 0  0  2 1  0 -3 0  1  5
por Laplace D = (-1)(-1)(-1)(1).
 0  2 1  5
D = 2 != 0
Logo não existe solução para o sistema
[]'sJoão
Date: Tue, 7 Feb 2012 17:09:26 -0300
Subject: [obm-l] Problema
From: bob...@globo.com
To: obm-l@mat.puc-rio.br

0lá ,

Poderiam me ajudar na questão  a seguir  ?

Em uma circunferência colocamos os números 5, 2,3,0, 5 e 6 (por exemplo nesta 
ordem  no sentido horário) .A cada momento escolho  um número qualquer e 
adiciono uma unidade a ele e aos dois vizinhos .É posível em algum momento 
todos ficarem iguais ? se possível , como devemos proceder ?


Agradeço desde já qualquer  resposta 

Bob 



  

RE: [obm-l] Problema

2012-02-07 Thread João Maldonado

Na  verdade foi besteira minha considerar toda essa solução,  há uma  maneira 
bem mais  fácil de provar que não existe solução
a-d = 1d-a = 1
somando ->0 = 2,  impossível
Logo não  existe solução para o sistema  


Devia  ter pensado nisso antes, hehe
Mas note que essa prática só vale para 6 números,  se tivéssemos uma 7ª  
incógnita, por exemplo,  já teríamos que fazer por Rouché-Capelli
[]'sJoão


From: joao_maldona...@hotmail.com
To: obm-l@mat.puc-rio.br
Subject: RE: [obm-l] Problema
Date: Tue, 7 Feb 2012 21:15:36 -0200







Sendo  "a" a quantidade de vezes que foi realizada a operação no número 5, "b"  
no 2, "c" no 3 ...
Montamos  o sistema
5+f+a+b = 2 + a + b + c = 3 + b + c + d = 0 + c + d + e = 5 + d + e + f = 6 + e 
+ f + a

que é  equivalente a:
a-d = 1b-e = -3c-f = 5d-a = 1e-b = -1f-c = -3

Podemos  facilmente resolvê-lo  por Rouché-Capelli
a matriz incompleta será: 
 1  0  0 -1  0  0 0  1  0  0 -1  0 0  0  1  0  0 -1-1  0  0  1  0  0 0 -1  0  0 
 1  0 0  0 -1  0  0  1
Somando a primeira linha com a quarta obtemos uma linha nula -> determinante 
igual a zeroEliminando a última linha e coluna:
 1  0  0 -1  0 0  1  0  0 -1 0  0  1  0  0-1  0  0  1  0 0 -1  0  0  1
Somando a primeira linha com a quarta obtemos uma linha nula -> determinante 
igual a zeroEliminando a última linha e coluna: 1  0  0 -1 0  1  0  0 0  0  1  
0-1  0  0  1
Somando a primeira linha com a quarta obtemos uma linha nula -> determinante 
igual a zeroEliminando a última linha e coluna:
 1  0  0 0  1  0 0  0  1
Temos determinante 1,  logo o determinante principal da matriz é 1
A  matriz tem 3 determinantes característicos
Analisando o primeiro
 1  0  0  1 0  1  0 -3 0  0  1  5-1  0  0  1
Somando a primeira coluna com a última:
 1  0  0  2 0  1  0 -3 0  0  1  5-1  0  0  0
por  Laplace  D = (-1)(-1).
 0  0  2 1  0 -3 0  1  5
por Laplace D = (-1)(-1)(-1)(1).
 0  2 1  5
D = 2 != 0
Logo não existe solução para o sistema
[]'sJoão
Date: Tue, 7 Feb 2012 17:09:26 -0300
Subject: [obm-l] Problema
From: bob...@globo.com
To: obm-l@mat.puc-rio.br

0lá ,

Poderiam me ajudar na questão  a seguir  ?

Em uma circunferência colocamos os números 5, 2,3,0, 5 e 6 (por exemplo nesta 
ordem  no sentido horário) .A cada momento escolho  um número qualquer e 
adiciono uma unidade a ele e aos dois vizinhos .É posível em algum momento 
todos ficarem iguais ? se possível , como devemos proceder ?


Agradeço desde já qualquer  resposta 

Bob 



  

RE: [obm-l] Problema

2012-02-07 Thread João Maldonado

Na  verdade eu não errei o sistema,  só  resolvi pelo método mais difícil, hehe
Mas  valeu pela dica
[]'sJoão

From: vitor__r...@hotmail.com
To: obm-l@mat.puc-rio.br
Subject: RE: [obm-l] Problema
Date: Wed, 8 Feb 2012 02:45:35 +0300








João Maldonado,não li sua solução por completo,mas vê-se facilmente que vc 
errou no sistema ou vc naum prestou atenção q naum seria necessário vc fazer o 
sistema,pois de a-d=1 e d-a=1 tem-se um absurdo.AbraçosVitor de Lima Alves
From: joao_maldona...@hotmail.com
To: obm-l@mat.puc-rio.br
Subject: RE: [obm-l] Problema
Date: Tue, 7 Feb 2012 21:15:36 -0200







Sendo  "a" a quantidade de vezes que foi realizada a operação no número 5, "b"  
no 2, "c" no 3 ...
Montamos  o sistema
5+f+a+b = 2 + a + b + c = 3 + b + c + d = 0 + c + d + e = 5 + d + e + f = 6 + e 
+ f + a

que é  equivalente a:
a-d = 1b-e = -3c-f = 5d-a = 1e-b = -1f-c = -3

Podemos  facilmente resolvê-lo  por Rouché-Capelli
a matriz incompleta será: 
 1  0  0 -1  0  0 0  1  0  0 -1  0 0  0  1  0  0 -1-1  0  0  1  0  0 0 -1  0  0 
 1  0 0  0 -1  0  0  1
Somando a primeira linha com a quarta obtemos uma linha nula -> determinante 
igual a zeroEliminando a última linha e coluna:
 1  0  0 -1  0 0  1  0  0 -1 0  0  1  0  0-1  0  0  1  0 0 -1  0  0  1
Somando a primeira linha com a quarta obtemos uma linha nula -> determinante 
igual a zeroEliminando a última linha e coluna: 1  0  0 -1 0  1  0  0 0  0  1  
0-1  0  0  1
Somando a primeira linha com a quarta obtemos uma linha nula -> determinante 
igual a zeroEliminando a última linha e coluna:
 1  0  0 0  1  0 0  0  1
Temos determinante 1,  logo o determinante principal da matriz é 1
A  matriz tem 3 determinantes característicos
Analisando o primeiro
 1  0  0  1 0  1  0 -3 0  0  1  5-1  0  0  1
Somando a primeira coluna com a última:
 1  0  0  2 0  1  0 -3 0  0  1  5-1  0  0  0
por  Laplace  D = (-1)(-1).
 0  0  2 1  0 -3 0  1  5
por Laplace D = (-1)(-1)(-1)(1).
 0  2 1  5
D = 2 != 0
Logo não existe solução para o sistema
[]'sJoão
Date: Tue, 7 Feb 2012 17:09:26 -0300
Subject: [obm-l] Problema
From: bob...@globo.com
To: obm-l@mat.puc-rio.br

0lá ,

Poderiam me ajudar na questão  a seguir  ?

Em uma circunferência colocamos os números 5, 2,3,0, 5 e 6 (por exemplo nesta 
ordem  no sentido horário) .A cada momento escolho  um número qualquer e 
adiciono uma unidade a ele e aos dois vizinhos .É posível em algum momento 
todos ficarem iguais ? se possível , como devemos proceder ?


Agradeço desde já qualquer  resposta 

Bob 



  

RE: [obm-l] Problema

2012-02-07 Thread João Maldonado

kkk
Realmente  eu não prestei atenção nessa parte :x
Valeu  Bernardo!

> Date: Wed, 8 Feb 2012 00:45:03 +0100
> Subject: Re: [obm-l] Problema
> From: bernardo...@gmail.com
> To: obm-l@mat.puc-rio.br
> 
> 2012/2/8 João Maldonado :
> > Sendo  "a" a quantidade de vezes que foi realizada a operação no número 5,
> > "b"  no 2, "c" no 3 ...
> >
> > Montamos  o sistema
> >
> > 5+f+a+b = 2 + a + b + c = 3 + b + c + d = 0 + c + d + e = 5 + d + e + f = 6
> > + e + f + a
> >
> > que é  equivalente a:
> >
> > a-d = 1
> > b-e = -3
> > c-f = 5
> > d-a = 1
> > e-b = -1
> > f-c = -3
> 
> Parô parô parô !!
> a - d = 1
> d - a = 1 (absurdo)
> 
> b - e = -3
> e - b = -1 (outro absurdo)
> 
> c - f = 5
> f - c = -3 (aiaiaiaiai...)
> 
> Genial, é assim que dá pra interpretar "a diferença dos 666 e o 462"
> que eu botei no mail anterior. Valeu João por montar o sistema e não
> ser preguiçoso como eu!
> 
> -- 
> Bernardo Freitas Paulo da Costa
> 
> =
> Instruções para entrar na lista, sair da lista e usar a lista em
> http://www.mat.puc-rio.br/~obmlistas/obm-l.html
> =
  

Re: [obm-l] Problema

2012-02-08 Thread Eduardo Wilner



A velocidade da nave que viaja pela diagonal eh o triplo da que viaja pela 
aresta, percorrendo uma diastancia \sqrt3 vezes a percorrida pela segunda, 
portanto num intervalo de tempo menor.
Como elas terminam as "viagens" no mesmo instante t=0, no instante t=-1 ( no 
exemplo da resolução ) , quando a nave "alfa" estah partindo de B, a nave 
"beta" ainda não está partindo ou estaria virtualmente num ponto aquem de A, no 
caso (1-\sqrt3)(1,1,1).

Me parece que o problema seria mais "Olímpico" (e talvez fosse a intenção do 
autor, que se distraiu) se a nave que viaja pela diagonal tivesse velocidade 
\srt3 vezes a que viaja pela aresta, (quando sua pergunta teria "a ver", como 
dizem os jovens) e a distancia seria minima no meio do percurso, igual a 
(sqrt2)/2... 

[ ]'s
 

--- Em dom, 5/2/12,
 Henrique Rennó  escreveu:

De: Henrique Rennó 
Assunto: [obm-l] Problema
Para: "obm-l" 
Data: Domingo, 5 de Fevereiro de 2012, 12:41

Oi, boa tarde.

A solução do problema 1 da primeira fase do nível universitário na eureka 34 
página 60 
(http://www.obm.org.br/export/sites/default/revista_eureka/docs/eureka34.pdf) 
apresenta a função de posição considerando tempos inicial -1 e final 0, sendo a 
função do objeto mais rápido dada por B(t) = (1,1,1) + rq(3)*t*(1,1,1), onde rq 
é a raíz quadrada. Considerando t = -1 na equação, temos B(-1) = (1-rq(3), 
1-rq(3), 1-rq(3)), que é
 diferente da posição inicial (0,0,0). Outra dúvida é como ficariam as funções 
se considerarmos como tempo inicial e final os valores 0 e 1, respectivamente.


Obrigado

-- 
Henrique




Re: [obm-l] Problema

2012-02-09 Thread Bernardo Freitas Paulo da Costa
2012/2/8 Bernardo Freitas Paulo da Costa :
> 2012/2/8 João Maldonado :
>> Sendo  "a" a quantidade de vezes que foi realizada a operação no número 5,
>> "b"  no 2, "c" no 3 ...
>>
>> Montamos  o sistema
>>
>> 5+f+a+b = 2 + a + b + c = 3 + b + c + d = 0 + c + d + e = 5 + d + e + f = 6
>> + e + f + a
>>
>> que é  equivalente a:
>>
>> a-d = 1
>> b-e = -3
>> c-f = 5
>> d-a = 1
>> e-b = -1
>> f-c = -3
>
> Parô parô parô !!
> a - d = 1
> d - a = 1 (absurdo)
>
> b - e = -3
> e - b = -1 (outro absurdo)
>
> c - f = 5
> f - c = -3 (aiaiaiaiai...)
>
> Genial, é assim que dá pra interpretar "a diferença dos 666 e o 462"
> que eu botei no mail anterior. Valeu João por montar o sistema e não
> ser preguiçoso como eu!
Pensando melhor na minha mensagem, isso dá o "invariante" que mata o problema!

Veja bem. Começando do 0 0 0 0 0 0, cada operação "somar três uns" vai
fazer +1 +1 +1 0 0 0 ou alguma permutação circular disso. Repare que,
para "tornar todos os números iguais", as operações +1 +1 +1 0 0 0 e 0
0 0 +1 +1 +1 são "inversas" uma da outra. Então na verdade temos 6
números, e podemos fazer 3 operações, ou "desfazer" essas três. Isso
dá um "espaço vetorial" (na verdade, apenas um reticulado, porque
temos apenas os pontos "inteiros"), e assim partindo do 0 0 0 0 0 0
podemos percorrer apenas um "espaço de dimensão 3". Ora, começamos com
um treco de dimensão aparentemente 6 (acho que vai dar 5, porque o que
importa são as diferenças, e a soma das diferenças cíclicas é zero),
portanto "a maior parte do tempo" não vai dar certo.

Voltemos à situação original com 5 2 3 0 5 6. Note que existe apenas
uma transformação que permite reduzir a diferença entre o 5 e o 2:
somar 1 na 2a, 3a e 4a casa. Então, temos que fazer 3 vezes essa
operação. Mas o problema é que isso vai dar 5 5 6 3 5 6, e 3 != 5 (a
4a casa diferente da 5a). Ora, para corrigir essa diferença, a gente é
obrigado (de novo) a usasr a


> --
> Bernardo Freitas Paulo da Costa



-- 
Bernardo Freitas Paulo da Costa

=
Instruções para entrar na lista, sair da lista e usar a lista em
http://www.mat.puc-rio.br/~obmlistas/obm-l.html
=


Re: [obm-l] Problema

2012-02-09 Thread Bernardo Freitas Paulo da Costa
E o GMail cortou a minha mensagem de graça...

2012/2/9 Bernardo Freitas Paulo da Costa :
> Pensando melhor na minha mensagem, isso dá o "invariante" que mata o problema!
>
> Veja bem. Começando do 0 0 0 0 0 0, cada operação "somar três uns" vai
> fazer +1 +1 +1 0 0 0 ou alguma permutação circular disso. Repare que,
> para "tornar todos os números iguais", as operações +1 +1 +1 0 0 0 e 0
> 0 0 +1 +1 +1 são "inversas" uma da outra. Então na verdade temos 6
> números, e podemos fazer 3 operações, ou "desfazer" essas três. Isso
> dá um "espaço vetorial" (na verdade, apenas um reticulado, porque
> temos apenas os pontos "inteiros"), e assim partindo do 0 0 0 0 0 0
> podemos percorrer apenas um "espaço de dimensão 3". Ora, começamos com
> um treco de dimensão aparentemente 6 (acho que vai dar 5, porque o que
> importa são as diferenças, e a soma das diferenças cíclicas é zero),
> portanto "a maior parte do tempo" não vai dar certo.
>
> Voltemos à situação original com 5 2 3 0 5 6. Note que existe apenas
> uma transformação que permite reduzir a diferença entre o 5 e o 2:
> somar 1 na 2a, 3a e 4a casa. Então, temos que fazer 3 vezes essa
> operação. Mas o problema é que isso vai dar 5 5 6 3 5 6, e 3 != 5 (a
> 4a casa diferente da 5a). Ora, para corrigir essa diferença, a gente é
> obrigado (de novo) a
usar a mesma operação (ou a sua "inversa", se a diferença fosse no
outro sentido), o que desequilibra as diferenças entre a 1a e 2a casa.
Portanto, é impossível.

Isso dá uma condição necessária para conseguir terminar: as diferenças
1a - 2a, 2a - 3a, 3a - 4a casas têm que ser iguais às diferenças 5a -
4a, 6a - 5a, 1a - 6a.
Assim, se tivermos
A B C D E F
no início, temos que ter
A - B = E - D
B - C = F - E
C - D = A - F
(que são, se você notar, os números que "dão errado" nas equações do João)
Veja que a soma das duas primeiras equações é A - C = F - D que é
equivalente à 3a equação, portanto só há mesmo 2 restrições (o que é
"coerente" com ser um espaço de 5 dimensões e haver 3 operações livres
: duas direções proibidas = 2 equações). Portanto, se fixarmos A , B ,
C e D temos E = D + (A-B) e F = A + (D-C)

A B C D (D + A - B) (A + D - C)

realizando A - B vezes a operação que soma 1 nas 2a, 3a e 4a casas, temos

A A (C + A - B) (D + A - B) (D + A - B) (A + D - C)

Cancelando os A (para simplificar)

0 0 (C - B) (D - B) (D - B) (D - C)

fazendo B - C vezes a operação que soma 1 nas 3a, 4a e 5a casas:

0 0 0 (D - C) (D - C) (D - C)

fazendo C - D vezes a operação que soma 1 nas 4a, 5a e 6a casas :

0 0 0 0 0 0

Todos são iguais ! Portanto, a condição é suficiente também!

Abraços,
-- 
Bernardo Freitas Paulo da Costa

=
Instruções para entrar na lista, sair da lista e usar a lista em
http://www.mat.puc-rio.br/~obmlistas/obm-l.html
=


Re: [obm-l] PROBLEMA

2012-12-07 Thread Rogerio Ponce
Ola' Luis e Bernardo,
a letra "d" (a funcao leva circunferencias em circunferencias) esta' errada
porque, de modo geral, leva circunferencias (nao paralelas ao plano
horizontal) em elipses.
[]'s
Rogerio Ponce

Em 5 de dezembro de 2012 20:52, Luís Lopes  escreveu:

>  Sauda,c~oes,
>
> Mais uma tentativa de ver esta mensagem chegar na lista.
>
> O Bernardo já respondeu numa troca de emails particular.
>
> Esta mensagem deve ser lida de baixo para cima.
>
> Abraços,
> Luís
>
>
> O arquivo está em
> http://www.escolademestres.com/download/questao_luis_lopes_20121205.pdf
>
> > Date: Wed, 5 Dec 2012 09:59:41 -0500
> > Subject: Re: [obm-l] RE: PROBLEMA
> > From: bernardo...@gmail.com
> > To: qed_te...@hotmail.com
> >
> > 2012/12/5 Luís Lopes :
> > > Oi Bernardo,
> > >
> > > Não consigo me comunicar com a lista. Mas pelo jeito a
> > > lista (você) recebe(u) minhas msgs.
> > Estranho.
> >
> > > Tenho recebido somente algumas msgs da lista. Num outro
> > > email (outra conta) acho que recebo todos. Pode ser problema
> > > com o hotmail também, sei lá.
> > Pode ser...
> >
> > > Tudo começou com o email abaixo, que mandei pra lista:
> > Pode ser uma questão de PDFs. Sei lá. Pode ser mesmo "esse PDF em
> particular".
> >
> > Para responder a pergunta: todas as afirmações são verdadeiras, é
> > claro. O problema da "d" é que a projeção estereográfica envia
> > círculos passando pelo polo Norte em retas do plano + ponto no
> > infinito. Mas como f está definida apenas na esfera - polo Norte,
> > nenhuma dessas circunferências serve, porque f não aplica o polo norte
> > em lugar nenhum. Logo, as circunferências que você *pode* usar são
> > realmente mandadas em círculos do plano.
> >
> > Ah, óbvio que isso é um problema de geometria complexa, mas fica muito
> > mais legal retirar todo o contexto e fazer uma questão absurda de
> > vestibular. Pena, né.
> > --
> > Bernardo Freitas Paulo da Costa
>
> --
> From: qed_te...@hotmail.com
> To: obm-l@mat.puc-rio.br
> Subject: FW: PROBLEMA
> Date: Mon, 3 Dec 2012 12:44:01 +
>
> Sauda,c~oes,
>
> Mais uma vez, peço a ajuda de vocês para uma resposta
> mais completa e interessante.
>
> Espero que o anexo passe. Se não, alguém poderia colocá-lo
> disponível em algum site?
>
> 
> Subject: PROBLEMA
> Date: Mon, 3 Dec 2012 08:17:24 -0300
>
> Prezado Luís,
>
> Gostaria de sua ajuda para o problema(conforme arquivo PDF anexo).
> Se possível, gostaria que justificasse porque os demais itens são
> verdadeiros.
>
> Abraços,
>
> FERNANDO FORTALEZA-CE.
> 
>
> PS: E eu gostaria de saber também por que o item falso é falso.
> Obrigado.
>
> Abraços,
> Luís
>
>


Re: [obm-l] PROBLEMA

2012-12-07 Thread Rogerio Ponce
Para fazer justica ao enunciado, leia-se "paralelas ao plano P" em vez de
"paralelas ao plano horizontal".

[]'s
Rogerio Ponce

Em 7 de dezembro de 2012 21:06, Rogerio Ponce  escreveu:

> Ola' Luis e Bernardo,
> a letra "d" (a funcao leva circunferencias em circunferencias) esta'
> errada porque, de modo geral, leva circunferencias (nao paralelas ao plano
> horizontal) em elipses.
> []'s
> Rogerio Ponce
>
> Em 5 de dezembro de 2012 20:52, Luís Lopes escreveu:
>
>>  Sauda,c~oes,
>>
>> Mais uma tentativa de ver esta mensagem chegar na lista.
>>
>> O Bernardo já respondeu numa troca de emails particular.
>>
>> Esta mensagem deve ser lida de baixo para cima.
>>
>> Abraços,
>> Luís
>>
>>
>> O arquivo está em
>> http://www.escolademestres.com/download/questao_luis_lopes_20121205.pdf
>>
>> > Date: Wed, 5 Dec 2012 09:59:41 -0500
>> > Subject: Re: [obm-l] RE: PROBLEMA
>> > From: bernardo...@gmail.com
>> > To: qed_te...@hotmail.com
>> >
>> > 2012/12/5 Luís Lopes :
>> > > Oi Bernardo,
>> > >
>> > > Não consigo me comunicar com a lista. Mas pelo jeito a
>> > > lista (você) recebe(u) minhas msgs.
>> > Estranho.
>> >
>> > > Tenho recebido somente algumas msgs da lista. Num outro
>> > > email (outra conta) acho que recebo todos. Pode ser problema
>> > > com o hotmail também, sei lá.
>> > Pode ser...
>> >
>> > > Tudo começou com o email abaixo, que mandei pra lista:
>> > Pode ser uma questão de PDFs. Sei lá. Pode ser mesmo "esse PDF em
>> particular".
>> >
>> > Para responder a pergunta: todas as afirmações são verdadeiras, é
>> > claro. O problema da "d" é que a projeção estereográfica envia
>> > círculos passando pelo polo Norte em retas do plano + ponto no
>> > infinito. Mas como f está definida apenas na esfera - polo Norte,
>> > nenhuma dessas circunferências serve, porque f não aplica o polo norte
>> > em lugar nenhum. Logo, as circunferências que você *pode* usar são
>> > realmente mandadas em círculos do plano.
>> >
>> > Ah, óbvio que isso é um problema de geometria complexa, mas fica muito
>> > mais legal retirar todo o contexto e fazer uma questão absurda de
>> > vestibular. Pena, né.
>> > --
>> > Bernardo Freitas Paulo da Costa
>>
>> --
>> From: qed_te...@hotmail.com
>> To: obm-l@mat.puc-rio.br
>> Subject: FW: PROBLEMA
>> Date: Mon, 3 Dec 2012 12:44:01 +
>>
>> Sauda,c~oes,
>>
>> Mais uma vez, peço a ajuda de vocês para uma resposta
>> mais completa e interessante.
>>
>> Espero que o anexo passe. Se não, alguém poderia colocá-lo
>> disponível em algum site?
>>
>> 
>> Subject: PROBLEMA
>> Date: Mon, 3 Dec 2012 08:17:24 -0300
>>
>> Prezado Luís,
>>
>> Gostaria de sua ajuda para o problema(conforme arquivo PDF anexo).
>> Se possível, gostaria que justificasse porque os demais itens são
>> verdadeiros.
>>
>> Abraços,
>>
>> FERNANDO FORTALEZA-CE.
>> 
>>
>> PS: E eu gostaria de saber também por que o item falso é falso.
>> Obrigado.
>>
>> Abraços,
>> Luís
>>
>>
>


RE: [obm-l] problema

2013-02-15 Thread João Maldonado

abc + bcd + cda + dab = abcd -> a(bcd -bc-bd-cd) = bcd
Logo, temos qualquer número é divisor do produto dos outros 3 (I)

Ao fatorarmos abcd em potência de primos, podemos obter 1, 2, ou 3 primos 
distintos
Se houvesse 4 primos, por (I), cada um dos números, a, b, c, d deveria ser 
múltiplo dos 4 primos simultaneamente, o que daria a soma <= 4/2.3.5.7 < 1

Além disso, pelo menos um dos números que é menor ou igual a 4
Se a=4, (a,b,c,d) = (4,4,4,4)

Se tivermos 3 primos distintos os 2 primeiros valores da soma em ordem 
decrescente são
1/2+1/3+1/5+1/30 > 1
1/2+1/3+1/7+1/42 = 1
Logo temos (2, 3, 7, 42), já que quaisquer outros números implicariam e soma<1

Se tivermos 2 primos:
Se a=2
1/b+1/c+1/d = 1/2 -> b<=6 -> (2,6,6,6) ou b=3, 4 ou 5
Se b=3 -> 1/c+1/d = 1/6 -> c>6 e c<=12
d=6+36/(c-6) -> c=8,9 ou 12
Isso dá (2,3,8,24), (2,3, 9,18), (2,3,12,12)

Se b=4,  c>4 e c<=8
d=4+16(c-4) -> c = 5, 6, 8
Isso dá (2,4,5,20),(2,4,6,12)

Se b=5, 1/c+1/d=3/10, logo c<10 -> c=8 (pois c deve ser múltiplo de 2 e/ou 5),  
sem soluçã

Se a=3, B<=9/2 -> b=4 ou 3
Se b=3, 1/c+1/d=3, c<=6,
d=3+9(c-3), c=4, 6
Isso dá (3,3,4,6), (3,3,6,6)

Se b=4, 1/c+1/d = 5/12, c<=24/5 -> c=4
Isso dá (3,4,4,6)

Se tivermos 1 primo
p=2 ou 3
Pois  se p fosse 5, soma mámixa = 4/5<1
a=2 ou 3  (como já visto)
Se a=2
1/b+1/c+1/d = 1/2, b<=6, logo b=4
1/c+1/d = 1/4 -> c=d=8 -> (2,4,8,8)
Se a=3
1/b+1/c+1/d = 2/3 -> b<=9/2, logo b=3
1/c+1/d = 1/3, absurdo

Logo temos: (4,4,4,4), (2,4,8,8), (3,4,4,6), (3,3,4,6), (3,3,6,6),  (2,4,5,20), 
(2,3,6,12), (2,3,8,24), (2,3,9,18), (2,3,12,12) e (2,3,7,42)

[]'s
João


Date: Fri, 15 Feb 2013 16:31:10 -0800
From: profgr...@yahoo.com.br
Subject: [obm-l] problema
To: obm-l@mat.puc-rio.br

Olá, companheiros!Um aluno me perguntou o seguinte:a 
<=b<=c<=d1/a+1/b+1/c+1/d=1Quantas quádruplas ordenadas (a, b, c, d) de naturais 
satisfazem a igualdade?Um abraço!Grego   

RE: [obm-l] problema

2013-02-15 Thread João Maldonado

abc + bcd + cda + dab = abcd -> a(bcd -bc-bd-cd) = bcd
Logo, temos qualquer número é divisor do produto dos outros 3 (I)

Ao fatorarmos abcd em potência de primos, podemos obter 1, 2, ou 3 primos 
distintos
Se houvesse 4 primos, por (I), cada um dos números, a, b, c, d deveria ser 
múltiplo dos 4 primos simultaneamente, o que daria a soma <= 4/2.3.5.7 < 1

Além disso, pelo menos um dos números que é menor ou igual a 4
Se a=4, (a,b,c,d) = (4,4,4,4)

Se tivermos 3 primos distintos os 2 primeiros valores da soma em ordem 
decrescente são
1/2+1/3+1/5+1/30 > 1
1/2+1/3+1/7+1/42 = 1
Logo temos (2, 3, 7, 42), já que quaisquer outros números implicariam e soma<1

Se tivermos 2 primos:
Se a=2
1/b+1/c+1/d = 1/2 -> b<=6 -> (2,6,6,6) ou b=3, 4 ou 5
Se b=3 -> 1/c+1/d = 1/6 -> c>6 e c<=12
d=6+36/(c-6) -> c=8,9 ou 12
Isso dá (2,3,8,24), (2,3, 9,18), (2,3,12,12)

Se b=4,  c>4 e c<=8
d=4+16(c-4) -> c = 5, 6, 8
Isso dá (2,4,5,20),(2,4,6,12)

Se b=5, 1/c+1/d=3/10, logo c<10 -> c=8 (pois c deve ser múltiplo de 2 e/ou 5),  
sem soluçã

Se a=3, B<=9/2 -> b=4 ou 3
Se b=3, 1/c+1/d=3, c<=6,
d=3+9(c-3), c=4, 6
Isso dá (3,3,4,6), (3,3,6,6)

Se b=4, 1/c+1/d = 5/12, c<=24/5 -> c=4
Isso dá (3,4,4,6)

Se tivermos 1 primo
p=2 ou 3
Pois  se p fosse 5, soma mámixa = 4/5<1
a=2 ou 3  (como já visto)
Se a=2
1/b+1/c+1/d = 1/2, b<=6, logo b=4
1/c+1/d = 1/4 -> c=d=8 -> (2,4,8,8)
Se a=3
1/b+1/c+1/d = 2/3 -> b<=9/2, logo b=3
1/c+1/d = 1/3, absurdo

Logo temos: (4,4,4,4), (2,4,8,8), (3,4,4,6), (3,3,4,6), (3,3,6,6),  (2,4,5,20), 
(2,3,6,12), (2,3,8,24), (2,3,9,18), (2,3,12,12) e (2,3,7,42)

[]'s
João


Date: Fri, 15 Feb 2013 16:31:10 -0800
From: profgr...@yahoo.com.br
Subject: [obm-l] problema
To: obm-l@mat.puc-rio.br

Olá, companheiros!Um aluno me perguntou o seguinte:a 
<=b<=c<=d1/a+1/b+1/c+1/d=1Quantas quádruplas ordenadas (a, b, c, d) de naturais 
satisfazem a igualdade?Um abraço!Grego  
 

RE: [obm-l] problema

2013-02-15 Thread João Maldonado

Tem muita coisa errada ainda:

1)  Se b=5, 1/c+1/d=3/10, logo c<10 -> c=8 (pois c deve ser múltiplo de 2 e/ou 
5),  sem soluçã
Corrija para c=5, d=10

2) 
Se tivermos 3 primos distintos os 2 primeiros valores da soma em ordem 
decrescente são
1/2+1/3+1/5+1/30 > 1
1/2+1/3+1/7+1/42 = 1
Logo temos (2, 3, 7, 42), já que quaisquer outros números implicariam e soma<1

Eles não estão na ordem decrescente :)
Na verdade esqueça os casos de 1,2,3 primos, vamos fazer tudo no caso mais 
geral (eu tentei simplificar mas parece que piorou)

3) 
Se tivermos 2 primos:
Se a=2
1/b+1/c+1/d = 1/2 -> b<=6 -> (2,6,6,6) ou b=3, 4 ou 5
Se b=3 -> 1/c+1/d = 1/6 -> c>6 e c<=12
d=6+36/(c-6) -> c=8,9 ou 12
Isso dá (2,3,8,24), (2,3, 9,18), (2,3,12,12)

Na verdade temos c=10 também

4) 
E o caso (2,6,6,6) que eu tinha achado e não pus no final

Corrijindo temos 14 casos, conforme o post do Bouskela

[]'s
João



From: joao_maldona...@hotmail.com
To: obm-l@mat.puc-rio.br
Subject: RE: [obm-l] problema
Date: Sat, 16 Feb 2013 02:57:59 -0200





abc + bcd + cda + dab = abcd -> a(bcd -bc-bd-cd) = bcd
Logo, temos qualquer número é divisor do produto dos outros 3 (I)

Ao fatorarmos abcd em potência de primos, podemos obter 1, 2, ou 3 primos 
distintos
Se houvesse 4 primos, por (I), cada um dos números, a, b, c, d deveria ser 
múltiplo dos 4 primos simultaneamente, o que daria a soma <= 4/2.3.5.7 < 1

Além disso, pelo menos um dos números que é menor ou igual a 4
Se a=4, (a,b,c,d) = (4,4,4,4)

Se tivermos 3 primos distintos os 2 primeiros valores da soma em ordem 
decrescente são
1/2+1/3+1/5+1/30 > 1
1/2+1/3+1/7+1/42 = 1
Logo temos (2, 3, 7, 42), já que quaisquer outros números implicariam e soma<1

Se tivermos 2 primos:
Se a=2
1/b+1/c+1/d = 1/2 -> b<=6 -> (2,6,6,6) ou b=3, 4 ou 5
Se b=3 -> 1/c+1/d = 1/6 -> c>6 e c<=12
d=6+36/(c-6) -> c=8,9 ou 12
Isso dá (2,3,8,24), (2,3, 9,18), (2,3,12,12)

Se b=4,  c>4 e c<=8
d=4+16(c-4) -> c = 5, 6, 8
Isso dá (2,4,5,20),(2,4,6,12)

Se b=5, 1/c+1/d=3/10, logo c<10 -> c=8 (pois c deve ser múltiplo de 2 e/ou 5),  
sem soluçã

Se a=3, B<=9/2 -> b=4 ou 3
Se b=3, 1/c+1/d=3, c<=6,
d=3+9(c-3), c=4, 6
Isso dá (3,3,4,6), (3,3,6,6)

Se b=4, 1/c+1/d = 5/12, c<=24/5 -> c=4
Isso dá (3,4,4,6)

Se tivermos 1 primo
p=2 ou 3
Pois  se p fosse 5, soma mámixa = 4/5<1
a=2 ou 3  (como já visto)
Se a=2
1/b+1/c+1/d = 1/2, b<=6, logo b=4
1/c+1/d = 1/4 -> c=d=8 -> (2,4,8,8)
Se a=3
1/b+1/c+1/d = 2/3 -> b<=9/2, logo b=3
1/c+1/d = 1/3, absurdo

Logo temos: (4,4,4,4), (2,4,8,8), (3,4,4,6), (3,3,4,6), (3,3,6,6),  (2,4,5,20), 
(2,3,6,12), (2,3,8,24), (2,3,9,18), (2,3,12,12) e (2,3,7,42)

[]'s
João


Date: Fri, 15 Feb 2013 16:31:10 -0800
From: profgr...@yahoo.com.br
Subject: [obm-l] problema
To: obm-l@mat.puc-rio.br

Olá, companheiros!Um aluno me perguntou o seguinte:a 
<=b<=c<=d1/a+1/b+1/c+1/d=1Quantas quádruplas ordenadas (a, b, c, d) de naturais 
satisfazem a igualdade?Um abraço!Grego  
 

RE: [obm-l] Problema

2013-03-22 Thread João Maldonado
Eu consegui fazer para o caso geral (M e Q pode estar em qualquer região do 
círculo, não apenas em regiões opostas determinadas por um diâmetro)
E a resolução ficou bem "feia"" também (tive que usar cálculo)

*Sendo P1 um ponto a uma distância x fixa do centro do círculo, qual a 
probabilidade de escolhermos outro ponto no círculo tal que a distância entre 
P1 e P2 seja menor que um?
Podemos tracejar um círculo de raio 1 em torno de P1. A intersecção desse 
círculo com o círculo original é a região dos pontos cuja a distância a P1 é < 
1.
A área dessa região sobre a área do círculo simboliza a probabilidade de 
escolhermos outro ponto P2 no círculo tal que a distância entre P1 e P2 seja 
menor que um.
A área pode ser facilmente calculada por matemática básica

A/Atotal = 1/Pi (2 ArcCos[x/2] - x sqrt (1- (x/2)²))

O "peso" dessa probabilidade é proporcional à área que ela ocupa (temos muito 
mais pontos a uma distância 1 do que a uma distância 1/2 por exemplo)
O peso vale 2 Pi x dx/Pi = 2 x dx
Integrando de 0 a 1

P = Integral[ 2 x dx/Pi (2 ArcCos[x/2] - x sqrt (1- (x/2)²))]  de 0 a 1

P = 58.6%

[]'s
João

From: bened...@ufrnet.br
To: obm-l@mat.puc-rio.br
Subject: [obm-l] Problema
Date: Fri, 22 Mar 2013 05:16:50 -0300

ProblemaDois pontos, M e Q, são escolhidos aleatoriamente num disco unitário, 
mas em regiões opostas, determinadas por um diâmetro AB. Qual é a probabilidade 
de que a distância entre M e Q seja menor do que 1? 


Re: [obm-l] problema

2013-12-13 Thread Artur Steiner
Como a exponencial é sempre positiva, não há solução negativa. Para x >= 0, 
definamos f(x) = 2^x - x, de modo que f(0) = 1 e f'(x) = 2^x ln (2) - 1. Como 
ln(2) > 0, f' é estritamente crescente, logo f é convexa. f' se anula em x* tal 
que 2^x* = 1/ln(2). Como ln(2) está em (0, 1), 1/ln(2) > 1 e x* > 0. Logo, f 
tem um mínimo global em x*, que está no eixo real positivo.

Temos que min f = f(x*) = 1/ln (2) - (ln(1/ln(2)))/ln(2) = 1/ln(2) + 
ln(ln2))/ln(2) = (1 + ln(ln2)))/ln(2)

Com uma planilha Excel, verifiquei que f(x*) > 0. Logo, esta equação não tem 
solução real.

Artur Costa Steiner

> Em 13/12/2013, às 18:17, "saulo nilson"  escreveu:
> 
> encontre todas as soluçoes de 2^x=x
> 
> -- 
> Esta mensagem foi verificada pelo sistema de antivírus e 
> acredita-se estar livre de perigo.

-- 
Esta mensagem foi verificada pelo sistema de antiv�rus e
 acredita-se estar livre de perigo.


=
Instru��es para entrar na lista, sair da lista e usar a lista em
http://www.mat.puc-rio.br/~obmlistas/obm-l.html
=


Re: [obm-l] problema

2013-12-14 Thread Bernardo Freitas Paulo da Costa
2013/12/13 Artur Steiner :
> Como a exponencial é sempre positiva, não há solução negativa. Para x >= 0, 
> definamos f(x) = 2^x - x, de modo que f(0) = 1 e f'(x) = 2^x ln (2) - 1. Como 
> ln(2) > 0, f' é estritamente crescente, logo f é convexa. f' se anula em x* 
> tal que 2^x* = 1/ln(2). Como ln(2) está em (0, 1), 1/ln(2) > 1 e x* > 0. 
> Logo, f tem um mínimo global em x*, que está no eixo real positivo.
>
> Temos que min f = f(x*) = 1/ln (2) - (ln(1/ln(2)))/ln(2) = 1/ln(2) + 
> ln(ln2))/ln(2) = (1 + ln(ln2)))/ln(2)
>
> Com uma planilha Excel, verifiquei que f(x*) > 0. Logo, esta equação não tem 
> solução real.
f(x*) = [1 + ln(ln(2)) ] / ln(2) é maior do que 0 <=> 1 + ln(ln(2)) >
0 <=> ln(ln(2)) > -1 <=> ln(2) > exp(-1) = 1/2.7818...

Exponencial de novo: sse 2 > exp(1/2.7181...), e basta ver que 2 >
exp(1/2) <=> 2 > 1 + 1/2 + 1/2*1/4 + 1/3!*1/8 + ..., o que é verdade
porque, sem os fatoriais no denominador, seria igualdade! (e todos os
termos são positivos)

Abraços,
-- 
Bernardo Freitas Paulo da Costa

-- 
Esta mensagem foi verificada pelo sistema de antivírus e
 acredita-se estar livre de perigo.


=
Instruções para entrar na lista, sair da lista e usar a lista em
http://www.mat.puc-rio.br/~obmlistas/obm-l.html
=


Re: [obm-l] Problema

2005-09-16 Thread Maur�cio

  Rejane,

  Acho que a maneira mais fácil é você calcular a
velocidade de trabalho dos dois. "A" pode fazer 1
trabalho em 12 horas, ou 1/12 trabalhos em 1 hora. "B"
faz 1 trabalho em 18 horas, ou 1/18 trabalhos em 1
hora. Os dois juntos fazem (1/12 + 1/18) trabalhos em
1 hora. Aí v. calcula quanto eles levam pra fazer um
trabalho inteiro.
  Outra maneira: A faz 3 trabalhos em 36 horas. B faz
2 trabalhos em 36 horas. Os dois juntos fazem 5
trabalhos em 36 horas. Se eles conseguem dividir o
trabalho sem problemas, levam 36/5 horas pra fazer 1
trabalho.

  Abraços,
  Maurício

> Bom dia,
> 
> Alguém pode me ajudar com esse probleminha?
> 
> Um datilografo A pode fazer um trabalho em 12 horas
> e o datilógrafo B, em 18 horas.  Os dois juntos,
> poderão realizar o mesmo trabalho no seguinte tempo:
> 
> a) 7h 12minb) 7h 30minc) 8h 24min   
> d) 8h 30mine) 9h 16min
> 
> Obrigada,
> 
> 




__ 
Yahoo! Mail - PC Magazine Editors' Choice 2005 
http://mail.yahoo.com
=
Instruções para entrar na lista, sair da lista e usar a lista em
http://www.mat.puc-rio.br/~nicolau/olimp/obm-l.html
=


Re: [obm-l] Problema

2005-09-16 Thread Paulo Cesar
Bom Dia,
 
Seja Va a velocidade com que A realiza o trabalho e Vb a velocidade com que B realiza o mesmo trabalho. Chamaremos de P o trabalho em questão. Daí:
Va.12 = P e Vb.18 = P --> Va = P/12 e Vb = P/18 (velocidade x tempo = trabalho realizado).
Queremos saber em quanto tempo os dois realizarão o mesmo trabalho juntos, ou seja, o valor de t tal que (Va + Vb).t = P. Substituindo, ficamos com (P/12 + P/18).t = P --> t = 36/5 ou t = 7,2 h ou ainda t = 7h 12min.

Gabarito: A
 
Abraços à todos da lista


Re: [obm-l] Problema

2005-09-17 Thread saulo nilson
vA= x/12=3x/36
vB=x/18=2x/36
 
o A deve pegar 3/5 do trabalho para os dois acabarem juntos.
t = 3/5 * 12 =7,2h=7h12min
 
 
On 9/16/05, Rejane <[EMAIL PROTECTED]> wrote:

Bom dia,
 
Alguém pode me ajudar com esse probleminha?
 
Um datilografo A pode fazer um trabalho em 12 horas e o datilógrafo B, em 18 horas.  Os dois juntos, poderão realizar o mesmo trabalho no seguinte tempo:
 
a) 7h 12minb) 7h 30min    c) 8h 24min    d) 8h 30min    e) 9h 16min
 
Obrigada,
 
 


Re: [obm-l] problema

2005-10-30 Thread Brunno Fernandes
Acho que a maneira mais facil de resolver essa questão, até por ser de
concurso era por substituicao mesmo
tente por 1,2,3. ate chegar ao valor de 756 reais
26x=756
x=36 metros


- Original Message -
From: "elton francisco ferreira" <[EMAIL PROTECTED]>
To: 
Sent: Sunday, October 30, 2005 8:23 PM
Subject: [obm-l] problema


Ao analisar notas fiscais de uma firma, o auditor
deparou-se com a seguinte situação:


QuantidadeMercadoria   Preço unitário   Total (R$)
*metros  cetim 21,00  *56,00

Não era possível ver o numero de metros vendidos, mas
sabia-se que era um numero inteiro. No valor total, so
xapareciam os dois ultimos dos três algarismos da
parte inteira. Com as afirmações acima, o auditor
concluiu que a quantidade de cetim, em metros,
declarada nessa nota foi:

16
26
36
46
48








___
Promoção Yahoo! Acesso Grátis: a cada hora navegada você
acumula cupons e concorre a mais de 500 prêmios! Participe!
http://yahoo.fbiz.com.br/
=
Instruções para entrar na lista, sair da lista e usar a lista em
http://www.mat.puc-rio.br/~nicolau/olimp/obm-l.html
=

=
Instruções para entrar na lista, sair da lista e usar a lista em
http://www.mat.puc-rio.br/~nicolau/olimp/obm-l.html
=


Re: [obm-l] problema

2005-10-30 Thread Angelo Barone Netto
Claro que 48 nao e a alternativa correta, pois 48*21 termina em 8.
Como 16*21=336,
 26*21=546,
 36*21=756 e
 46*21=966, a unica possibilidade e 36.

Angelo Barone Netto <[EMAIL PROTECTED]>
=
Instruções para entrar na lista, sair da lista e usar a lista em
http://www.mat.puc-rio.br/~nicolau/olimp/obm-l.html
=


Re: [obm-l] Problema

2006-01-02 Thread Eduardo Wilner
 Prezado Garcia  Achei o problema interessante mas, não conhecendo as velhas  versões que vc. menciona,  não sei se é este o espírito da  questão. Por favor corrija.  O primeiro matemático  recebe o produto como sendo 4324 que  pode ser fatorado como 2*2*23*47, sendo sua dúvida como agrupar em dois  fatores.     Assim ele declara que o produto é insuficiente para se conhecer os dois fatores.    O segundo recebe a soma como 139 sabendo então que as paridades  dos dois números não são iguais, logo o produto seria par, e o primeiro  não saberia se é um par vezes um impar ou um par vezes um par, e  declara que já sabia  que o produto seria insuficiente.    O primeiro então sabe que a soma é impar e que os números são de  pardiade diferente: 2*2*23=92 e 47  . Declara : "Então conheço os  números"    O segundo (que não é bobo) faz o mes!
mo
 raciocínio que nós estamos fazendo e declara: "Nesse caso, eu também".    Seria isso, Garcia?  []sWilner        [EMAIL PROTECTED] escreveu:  Me lembrei de outro velho problema que me passaram com dados novos:Um gênio matemático recebe, num papel, a soma de dois números inteiros entre 2 e100. Um outro gênio recebe o produto dos mesmos dois números. Os dois iniciam odiálogo:   - Este produto não é o suficiente para achar os dois números.- Eu sabia.- Então, eu conheço estes números.- Nesse caso, eu também.- Quais são os dois números?Citando Adriano Torres :> Olá, sou novo aqui na lista, e gostaria de propor um problema para que me > ensinassem a solução.> Estou envian!
do a
 figura do triângulo para que possa ser visto.> É um triangulo isóceles, com AB = AC, ângulo bÂc = 20°, cBt = 30° e bTc > reto. Determinar o angulo cPq. Ficarei grato se souber a solução, há muito > tento e nao consigo resolver.> Desculpa pela má qualidade da imagem, a fiz no paint, nao tenho muita > habilidade.> Obrigado,>  Adriano Torres> > > => Instruções para entrar na lista, sair da lista e usar a lista em> http://www.mat.puc-rio.br/~nicolau/olimp/obm-l.html> => This message was sent using IMP, the Internet Messaging Program.=Instruções para entrar na list!
a, sair
 da lista e usar a lista emhttp://www.mat.puc-rio.br/~nicolau/olimp/obm-l.html=
		 
Yahoo! doce lar. Faça do Yahoo! sua homepage.

RE: [obm-l] Problema

2006-01-02 Thread João Gilberto Ponciano Pereira
Acho que o interessante do problema é chegar nesses números (92 e 47). Até o 
momento, a única coisa que consegui concluir é que a soma dos números não pode 
ser par, ou seja, um dos números é par e o outro é ímpar.
 
 
 -Original Message-
From: [EMAIL PROTECTED] [mailto:[EMAIL PROTECTED] Behalf Of Eduardo Wilner
Sent: Monday, January 02, 2006 12:51 PM
To: obm-l@mat.puc-rio.br
Subject: Re: [obm-l] Problema





 Prezado Garcia

  Achei o problema interessante mas, não conhecendo as velhas versões que vc. 
menciona,  não sei se é este o espírito da questão. Por favor corrija.

  O primeiro matemático  recebe o produto como sendo 4324 que pode ser fatorado 
como 2*2*23*47, sendo sua dúvida como agrupar em dois fatores. 
  Assim ele declara que o produto é insuficiente para se conhecer os dois 
fatores.
  O segundo recebe a soma como 139 sabendo então que as paridades dos dois 
números não são iguais, logo o produto seria par, e o primeiro não saberia se é 
um par vezes um impar ou um par vezes um par, e declara que já sabia  que o 
produto seria insuficiente.
  O primeiro então sabe que a soma é impar e que os números são de pardiade 
diferente: 2*2*23=92 e 47  . Declara : "Então conheço os números"
  O segundo (que não é bobo) faz o mesmo raciocínio que nós estamos fazendo e 
declara: "Nesse caso, eu também".
  Seria isso, Garcia?

  []s

Wilner 

 
[EMAIL PROTECTED] escreveu: 


Me lembrei de outro velho problema que me passaram com dados novos:

Um gênio matemático recebe, num papel, a soma de dois números inteiros entre 2 e
100. Um outro gênio recebe o produto dos mesmos dois números. Os dois iniciam o
diálogo:

- Este produto não é o suficiente para achar os dois números.
- Eu sabia.
- Então, eu conheço estes números.
- Nesse caso, eu também.
- Quais são os dois números?





Citando Adriano Torres :

> Olá, sou novo aqui na lista, e gostaria de propor um problema para que me 
> ensinassem a solução.
> Estou enviando a figura do triângulo para que possa ser visto.
> É um triangulo isóceles, com AB = AC, ângulo bÂc = 20°, cBt = 30° e bTc 
> reto. Determinar o angulo cPq. Ficarei grato se souber a solução, há muito 
> tento e nao consigo resolver.
> Desculpa pela má qualidade da imagem, a fiz no paint, nao tenho muita 
> habilidade.
> Obrigado,
> Adriano Torres
> 
> 
> =
> Instruções para entrar na lista, sair da lista e usar a lista em
> http://www.mat.puc-rio.br/~nicolau/olimp/obm-l.html
> =
> 





This message was sent using IMP, the Internet Messaging Prrogram.
=
Instruções para entrar na lista, sair da lista e usar a lista em
http://www.mat.puc-rio.br/~nicolau/olimp/obm-l.html
=





  _  

Yahoo! doce lar. Faça  
<http://us.rd.yahoo.com/mail/br/tagline/homepage_set/*http://br.yahoo.com/homepageset.html>
 do Yahoo! sua homepage.


=
Instruções para entrar na lista, sair da lista e usar a lista em
http://www.mat.puc-rio.br/~nicolau/olimp/obm-l.html
=


  1   2   3   4   5   6   7   8   9   10   >